Вы находитесь на странице: 1из 29

Soal Kidney – Mungud  Which of the following is the most likely

appropriate for restoring the diuretic response?


1. A 42 years old male comes to the Siloam hospital A. add with hydrochlorothiazide
with muscular weakness, mild headache after B. add with metolazone
eating salted fish. Usually he drinks only small C. replace with furosemide
amount of water. He said that usually his blood D. replace with hydrochlorothiazide
pressure is 100/70 mmHg. His blood pressure E. increasing the dose of chlorthalidone
140/90 mmHg. The doctor sends him to the
laboratory. The result shows serum sodium is 150 7. A 75-year-old woman with hypertension is being
mEq/L (N: 135 - 155), potassium is 3.5 mEq/L (N: treated with thiazide. Her blood pressure is
3.5 – 5.0), random blood glucose 150 mg/dl (N decrease to 120/76 mm Hg. After several months
<140). on the medication, she complains of being tired
In this condition, what will the kidney do? and weak. Which of the following substance in
A. Increase glomerular filtration rate analysis of blood indicates low values?
B. Increase obligatory water reabsorption A. calcium
C. Increase facultative water reabsortion B. uric acid
D. Increase renin secretion C. potassium
E. Increase aldosteron secretion D. sodium
E. glucose
2. To cover the above function, what mechanism is
used? 8. A jaundiced 1-week premature infant with elevated
A. Increase cardiac output free bilirubin is seen in the premature baby
B. Increase frequency of the heart nursery. The mother had received antibiotic
C. Increase secretion of renin combination for a urinary tract infection (UTI) 1
D. Increase secretion of antidiuretic hormone week before delivery.
E. Increase secretion of aldosteron Which of the following is the most likely cause of
the baby’s kernicterus?
3. Which factor support the above mechanism? A. cefixime
A. Filtration rate of the glomeruli B. amoxicillin
B. Active transport of the tubular cell C. azithromycin
C. High osmolality of the interstitium of medulla D. erythromycin
E. cotrimoxazole
D. Obligatory water reabsortion
E. Osmolality of the filtrate in distal convoluted 9. A 4-year-old-boy was brought to hospital with
tubule puffiness around the eyes, especially in the
morning and swelling over the legs. His mother
4. If the urinalysis is also done, what will you see also complaint he has foamy urine. Urine sample
about the glucose in urine? shows protein ++++ (>3.5 per 1.73 m 2 per 24
A. positive ++++ hour), albumin level 1.5g/dL (normal=3.5-5g/dL).
B. positive +++ What is the principal management in this patient?
C. positive ++ A. Albumin
D. positive+ B. Furosemide
E. negative C. Prednisone
D. Thiazide
5. In a kidney laboratory, using a micropipette, we E. Cyclosporin
can measure the hemodynamic pressure of
afferent arteriole, pressure in Bowman capsule, 10. A 13 year-old-girl was brought to emergency unit
and osmotic pressure in glomerular lumen. If the with seizure. Her mother complaint she had fever 2
pressure in afferent arteriole is 40 mmHg, in weeks ago, but has recovered. Now she has
Bowman capsule 1 mmHg, the osmotic pressure swelling around the eyes and legs and she has
of glomerulus is 30 mmHg, the effective filtration dark urine. From physical exam, she is
pressure in glomerulus is: (40-1-30) unconscious. Her blood pressure is 180/120
A. 71 mmHg mmHg. Urine sample shows erythrocytes 20/hpf
B. 69 mmHg and protein +. Albumin level within normal limit.
C. 11 mmHg What other laboratory findings to confirm the
D. 9 mmHg diagnosis in this patient?
E. – 9 mmHg A. Decreased anti streptolisin O (ASO),
increased DNase-B, decreased C3
6. A 63-year-old-man, with a history of heart failure B. Increased anti streptolisin O (ASO),
and edema fails to respond adequately to decreased DNase-B, decreased C3
maximum recommended dosages of C. Decreased anti streptolisin O (ASO),
chlorthalidone. increased DNase-B, increased C3

1
D. Increased anti streptolisin O (ASO), D. Complicated urinary tract infection
decreased DNase-B, increased C3 E. Chronic urinary tract infection
E. Increased anti streptolisin O (ASO)
(untuk faringitis), increased DNase- A 55-year-old man refers to ICU after the procedure
B(pd nefritik/strepto), decreased C3 open reduction of femur fracture and bleeding intra
operative. At the third day he became apatis, fever
11. A 70-year-old male with tuberculosis, he was 38,5 C with blood pressure 110/60 mmHg, pulse 55
treated with daily streptomycin for 1 week. After x/min, respiratory 20x/min, and urine output decreased
the last injection, his urine output diminished and to 10 cc/hour in 24 hours. At laboratory examination
the laboratory test revealed ureum 80 (20 – 40) findings creatinine 4 mg/dL (N: 0.5-1.5), ureum 100
mg/dL, creatinine 3.0 (0.5 - 1.5) mg/dL. (AIN ada mg/dL (N: 20-40), sodium 130 mEq/L (N: 135-155),
proteinuria, ATN ga daa proteinuria) potassium serum 7.5 mEq/L (N: 3.5-5.5), blood sugar
What is the most likely diagnosis in this patient? 200 mg/dL (N: <140).
From blood gas analysis finding pH 7.1, pCO2 30
A. Urinary tract infection mmHg, pO2 90 mmHg,
B. Sepsis HCO3 18 mEq/L. ECG examination finds widening of
C. Glomerulonephritis QRS wave.
D. Chronic renal failure
E. Acute tubular nephritis 15. What is the most critical condition can cause life
threatening the patient?
12. What is the most possible cause in this patient? A. nothing for awareness
(AIN hypersentivity, ATN ischemic ata toxin) B. ureum and creatinine at very high lvl
A. Renal ischemia C. hyperkalemia (CKD, AKI)
B. Direct toxic injury D. metabolic acidosis
C. Complex immune response E. severe hyponatremia
D. Urinary obstruction
E. Drug hypersensitivity 16. What is the treatment for the critical condition
above?
13. A 3 year old boy was brought to emergency unit A. Waiting for the general condition of the
with severe diarrhea since three days ago. On patient became more better
arrival he is not fully alert. His vital sign are as B. Give diuretic until the urine output will
follows: blood pressure is 80/60 mmHg, pulse is be normally
not palpable, respiratory rate was 36 breaths per C. Hemodialysis immediately
minute, and body temperature was 38.4 oC. On (hiperkalemi parah)
physical exam there is sunken anterior fontanel, D. Give sodium bicarbonate until pH of
dry mucous membranes, sunken eyes, lack of blood became normal
tears, poor skin turgor, and the capillary refill more E. Intubation and ventilator to maintain
then 2 seconds. The laboratory test in this patient the respiratory
reveal haemoglobin 13 g/dL(11.5-15.5 g/dL),
hematocrit 46% (35-45%), WBC 7,000/mm 3(5,500-
15,500/mm3), platelets count 260,000/mm3
3
(150,000-400,000 mm ), urea nitrogen 40 mg/dL 17. A 36-year-old male truck driver is referred to you
(5-18 mg/dL), and creatinine 1.7 mg/dL (0.3-0.7 for evaluation of recurrent urolithiasis. The patient
mg/dL). Urine specific gravity is 1.040 (N: 1.003 – has passed more than 20 calcium oxalate stones
1.030) since age 18. Previous therapy with
What is the most likely diagnosis in this patient? hydrochlorothiazide and cellulose phosphate has
A. Acute tubular nephritis been ineffective. Physical examination is normal.
B. Prerenal acute kidney injury Laboratory finding: Serum creatinine 1.0 mg/dL,
C. Intrinsic acute kidney injury Serum uric acid 8.9 mg/dL, Urine pH 5.0
D. Postrenal acute kidney injury Radiographs of the abdomen show smooth renal
E. Chonic renal failure silhouettes measuring 13.6 cm on the right and
14.2 cm on the left, and no radioopaque at kidney
14. A 10 year old girl was brought to your clinic with region. From sonography findings multiple
dysuria since 5 days ago. On physical exam she is hyperechoic shadow from both kidney with
healthy with no significant findings. From urine measure about 2 – 5 mm.
culture is found Eschericia coli wih CFU ≥ 100,000. Which of the following treatment is the most
What is the most likely diagnosis in this patient? effective for this patient?
A. Dietary sodium restriction
A. Acute pyelonephritic B. Dietary calcium supplementation
B. Reflux nephropathy C. High fluid intake and long-term urine
C. Uncomplicated urinary tract acidification
infection D. High fluid intake and allopurinol(untuk
asam urat=radiolucent)
2
E. Shock wave lithotripsy A. Wilms tumor
B. Renal Cell Carcinoma
18. A 75 year old man had urine incontinence since 3 C. Transitional carcinoma
months ago. He had been diagnosed dementia by D. Squamous carcinoma
the doctor since 6 months ago. On examination he E. Undifferentited carcinoma
had no prostate problems. What is the cause of his
urination’s problem? 23. A 60-year-old male experiences the onzet of
A. Impaired frontal inhibition headache, naussea ,vomitting fo 1 month. Physical
B. Impaired pontine inhibition examination reveals hypertention, subfebril.
C. Sympathetic system dysfunction Pyelogram showed the affected kidney
D. Parasympathetic system dysfunction assymmetrically contracted with deformity of the
E. Sensoric system dysfunction calyceal system. Gross of the affected kidney
showed contracted and has irregular granular
19. A 44 year old man was hospitalized for systemic surface.The parenchyma is atrophic and replace
fungal infection. He was on Amfoterisin B by fibrosis. Microscopic showed glomerular
treatment for 7 days. On examination, he is sclerosis, hyalinization and atrophi in cortical area.
conscious, BP 120/80 mmHg, pulse 80 x/min, RR Another areas showed fibrosis, chronic
16 x/min, temperature 36,7 C. Lab: Hb 13 gr%, inflammatory cells with limpocytes agregation.
HT 37%, WBC 8600, Platelet 340.000. Ureum 91, What is the most likely lesions occur in this
creatinin 2, 0. Volume urine 800 cc/24 jam kidney?
What is the patognomonic laboratory finding in this A. Renal cell carcinoma
case? B. Chronic pyelonephritis
A. Increased osmolality C. Acute nephritis
B. Leucocyturia D. Nephrotic syndrome
C. Muddy brown cast E. Diabetic nephropathy
D. Erythrocyte cast
E. Haemoglobinuria 24. A 5-year-old boy child who had a palpable
abdominal mass brought to hospital by his mother.
20. A 24-year-old woman went to a physician because The child later had abdominal distention from
pain during urination and frequency. On bowel obstruction. An USG reveals a 6 cm left
examination, she had has suprapubic pain, but no renal mass with necrosis and hemorrhage.
fever. Her urine was cloudy and shows Microscopic showed embryonal tissue with
microscopic evidence of erythrocytes, pyuria, and abortive of tubular and glomerular structures.
gram-positive bacteria. In the past medical history, What is the diagnosis ot this patient?
she never had any symptoms like this one. Which A. Teratoma
organism is most likely responsible for this B. Retinoblastoma
patient’s illness? C. Wilms’tumor
A. Klebsiella pneumonia D. Hanarthoma
B. Pseudomonas aeruginosa E. Hemangioma

C. Escherichia coli 25. A 62-year-old woman presents with hematuria and


D. Proteus mirabilis left flank pain. Physical examination reveals
E. Staphylococcus saprophyticus costovertebral angle tenderness. Renal ultrasound
shows marked left hydronephrosis. Serum
21. A 55-year-old male complains hematuria at the creatinine and CBC are normal.
physician’s office for 2 weeks. A cystoscopic Which of the following diagnostic test is the most
examinations showed a 2 cm,exophytic,friable relevant?
mass on the right bladder dome. A. Renal biopsy
A biopsy of this lesion show findings most B. Renal arteriogram
consistent with a/an C. Palpation for inguinal lymph nodes
A. Papilloma D. Antinuclear antibody test
B. Verrucous carcinoma E. IVP
C. Transitional carcinoma
D. Squamous carcinoma 26. A 34-year-old man comes with 3-day history of low
E. Adenocarcinoma back pain, urinary hesitancy, and pain with
urination. He has not had any urethral discharge
22. A 50-year-old male presents with a month long or recent sexual contacts. His temperature is
history of flank pain and hematuria.An USG 38.2C,Z
abdominal results a 3 cm bosselated mass,
protrudes from cortical are in the right 27. A 25-year-old man has been crashed by a truck.
kidney.Cytologic examination reveals malignant The doctor suspects that his kidney is rupture, and
cells. then he sent the patient to have IVP.
What is most likely biopsy finding for this patient?
3
What is the most likely the appearance of renal normal; and he appears to be in pain. A urinalysis
parenchymal rupture in IVP? shows erythrocytes, leukocytes esterase and a
A. The kidney shows non function high pH.
B. Ekstravasation (urine bocor ) Which antigenic composition is most likely
C. Hydronephrosis (urine akumulasi di responsible for this illness?
renal pelvis) A. Hemolysin (proteus)
D. Normal nephrogram B. Catalase
E. Dilatation of calyx C. Fibronectin
D. Opa protein
28. A 50-year-old man comes to the clinic with E. Cord factor
complain of abdominal colic. The result of IVP
demonstrates ureteric dilatation. 33. A 67-year-old man with an episode of acute urine
What is the most likely problem? retention was catheterized. Three days later, he
A. Stone in the pelvicureter junction developed fever and suprapubic pain. Culture of
B. Kidney cyst urine revealed a thin film of bacterial growth over
C. Stone in the vesicoureter junction the entire blood agar plate, and urease test was
D. Renal cell Ca stage I positive.
E. Acute inflammation Which of the following is most likely organism to
cause this infection?
29. A 24-year-old woman went to a physician because A. Escherichia coli
of pain during urination. On examination, she had B. Staphylococcus sapropyticus
has left flank tenderness and low-grade fever. Her C. Proteus mirabilis
urine was cloudy and shows microscopic evidence D. Enterococcus faecalis
of erythrocytes, pyuria, and gram-positive bacteria. E. Morganella morgagni
In the past medical history, she never had any
symptoms like this one. 34. A 34-year-old woman presents with pain during
Which organism is most likely responsible for this urination and frequency. On examination, she had
patient’s illness? has suprapubic pain, but no fever. Her urine was
A. Klebsiella pneumonia cloudy and shows microscopic evidence of
B. Pseudomonas aeruginosa erythrocytes, pyuria, and bacteria. The urine has a
C. Escherichia coli high pH. Which of the following bacteria’s strategy
D. Proteus mirabilis is responsible for this condition?
E. Staphylococcus saprophyticus A. Destroy blood vessel in the kidney
B. Exhibits “swarming” motility
30. Which urine specimen is most appropriate for the C. Ferments many sugars
culture examination? D. Produce a potent urease
A. Suprapubic aspiration E. Secretes many exotoxins
B. Mid stream
C. Straight catheter 35. IVP image of a new born baby shows that left
D. Indwelling catheter kidney tremendously enlarged and clearly
E. 24 hours urine collection demonstrates intra-parenchyma mass. What
abnormality could possibly occur?
31. A 30-year-pregnant woman was complaining fever, A. horseshoe kidney
pain when urinating, blood in her urine, and severe B. pelvic kidney
lower back pain. The physician submits her urine C. unilateral renal agenesis
for culture; the next day, the laboratory reports an D. polycystic kidney (ada massa)
organisms growth count of greater than 105/ml of E. pancake kidney
urine. The organism grew well on the blood agar
and producing β–hemolytic colonies. 36. Oligohydramnios can be related to malformation in
Which important virulence factor for the organism kidney. Why can the amount of amniotic fluid
is most likely responsible for this patient’s illness? determine such malformation?
A. Capsule A. baby drinks the amniotic fluid
B. P-fimbriae (Ecoli) B. baby cannot drink amniotic fluid
C. Endotoxin C. amniotic fluid mostly contains urine
D. Phospolipase C D. little amount of amniotic fluid causes
E. Lipooligopolysaccharide the pelvic malformation
E. little amount of amniotic fluid disturbs
32. A 30-year-old man patient complained of a sharp the kidney development
pain in his loin radiating down to his perineum,
accompanied by hematuria. He reported that he 37. A new born baby boy has a malformation in
had a few days of urinary urgency, frequency, and genitourinary appearance. When he urinates, the
burning when urinating. On examination, his urine comes out from large area below the urethra
temperature was 37.5ºC; other vital sign were
4
opening. What process could possible failed during E. Steven Johnson Syndrome
the development of genitourinary process?
A. closure of genital fold
B. closure of genital opening 42. A 61-year-old man comes to the physician
C. closure of genital tubercle because of a 3-month history of episodes of
D. closure of urogenital sinus headache, palpitation, and excessive sweating. He
E. closure of Wollfian duct has had a 10-kg (22-lb) weight loss during this
period. While being examined, during an episode
38. Cryptorchid condition results in germ cells failed to his blood pressure is 210/110 mm Hg. Physical
multiply, and then die and only in Sertoli cells exist examination shows no other abnormalities. Urine
in seminferous tubules. What is the development studies show increased catecholamine
process the main cause of cryptorchid? concentrations. A CT scan of the abdomen is most
A. inguinal hernia likely to show a mass in which of the following
B. failure of testicular decent locations?
C. failure of gubernaculums regression A. Adrenal glands (aldosterone dan
D. fusion of the tunica albuginea and catecolamin dan adrenalin)
peritoneum B. Ureter
E. guevedoces C. Urethral
D. Bladder
39. A 30-year-old man had nausea, vomiting, and E. Right kidney
severe colicky right flank pain radiating into the
thigh for 4 hours. He is afebrile. There is right 43. A 6-year-old boy has a large intra-abdominal mass
costovertebral angle tenderness. Urinalysis shows in the midline just above the symphysis pubis.
numerous RBCs and no bacteria. During an operation, a cystic mass is found
Which of the following is the most likely diagnosis? attached to the umbilicus and the apex of the
A. Acute glomerulonephritis bladder. Which of the following is the most likely
B. Bacterial cystitis diagnosis?
C. Benign prostatic hyperplasia A. Hydrocele
D. Urolithiasis B. Meckel cyst
E. Renal cell carcinoma C. Meckel diverticulum
D. Omphalocele
40. A previously healthy 72-year-old man comes to the E. Urachal cyst
physician because of decreased urinary output
during the past 2 days; he has had no urinary 44. A 69-year-old man with 82 kg is referred to you
output for 8 hours. Examination shows suprapubic with fatigue and laboratory result shows Hb 7 g/dL,
fullness and enlarged prostate. ureum 55 mg/dL, and creatinin 5.4 mg/dL. He has
Which of the following is the most appropriate next not seen a physician for several years.
step in management? Which finding would help you decide that this
A. PSA examination condition is chronic?
B. Ultrasonography of the kidneys A. Low serum calcium
C. CT scan of the abdomen B. Low serum bicarbonate
D. Bladder catheterization C. Protein detected in urinalysis
E. Intravenous pyelography D. Broad waxy casts in urinalysis
E. Elevated serum phosphorus
41. An otherwise healthy 20-year-old woman is given
sulfamethoxazole to treat a bladder infection. 45. A 52-year-old woman with longstanding type II
Three days after beginning the antibiotic regimen, diabetes is referred with microalbuminuria and
the patient has moderately severe jaundice and normal serum creatinine. Her weight is 85 kg and
dark urine. Pain with urination and a low-grade height is 160 cm. Blood pressure is 150/100
fever have resolved. Her hematocrit is 20%. mmHg.
Substantial numbers of erythrocytes contain Heinz Which of the following measures should you
bodies. Her condition worsens until day 6 of advise to prevent progression to overt
antibiotic therapy, when it begins to resolve. nephropathy?
Symptoms are completely gone by day 9 of A. Tight glucose control
continued antibiotic therapy. B. High-protein and low carbohydrate
Which of the following conditions is the most likely diet
explanation for these findings? C. Blood pressure control
A. Aplastic anemia D. Regular diuretic
B. Generalized cytochrome-b5 reductase E. Increased water intake
deficiency
C. Glucose-6-phosphate dehydrogenase 46. A 20-year-old man presents with malaise, nausea,
deficiency and decreased urine output. He was previously
D. Pyruvate kinase deficiency well and his physical examination is normal except
5
for an elevated jugular venous pressure (JVP) and
pericardia rub. His electrolytes reveal acute renal 50. A 6-year-old girl, brought by her mother to the
failure (ARF). clinic with complaints of swelling that begins from
Which of the following findings on the urinalysis is both of eyelid throughout her body. Blood tests
most likely in keeping with acute obtained: albumin 2 g / dL, total cholesterol 250
glomerulonephritis? mg / dL. On examination of urine obtained protein
A. Granular casts (dan epithelial = ATN) +3. Which of the following is the most likely
B. Erythrocyte casts diagnosis for the case above?
C. Proteinuria A. Nephritic syndrome
D. Hyaline casts (prerenal) B. Nephrotic syndrome
E. White blood cell casts C. Acute renal failure
D. Chronic renal failure
47. A 35-year-old women presents with nausea and E. Glomerulonefritis
vomiting of 2 days duration. She is not on any 51. According to the case above, which of the
medications and was previously well until now. Her following is the most likely pathophysiology that
physical examination is normal except for postural caused of swelling?
drop in her blood pressure from 110/80 mmHg A. Infection
supine to 90/80 mmHg standing. Her serum B. Hypercholesterolemia
electrolytes are Sodium 130 mEq/L, Potassium 3 C. Hypoalbuminemia
mEq/L, Chloride 90 mEq/L, bicarbonate 30 mEq/L, D. Proteinuria
urea 50 mg/dL, and creatinine 0.8 mg/dL. E. Water retention
Which of the following electrolytes in the case
above is most likely to be filtered through the 52. A 39-year-old woman with a history of recurrent
glomerulus but unaffected by tubular secretion? urinary tract infections and now suffering that
A. Potassium infection. Culture examination of urine samples
B. Urea showed Escherichia colli as the cause of infection.
C. Creatinine Patients were given doses of ciprofloxacin in the
D. Bicarbonate treatment and symptoms of infection disappear.
E. Sodium Which of the following is the most correct process
or enzyme that inhibited by the drug?
48. A 68-year-old man came seeing a general A. Cell membrane synthesis
practitioner with a swelling throughout the body. B. Cell protein synthesis
He also felt urine volume decreases and the urine C. Dehidrofolate sintetase
color was red. He denied any pain on urination or D. DNA gyrase
in any part of his body, on physical examination E. DNA polimerase
costovertebrae angular test negative. Blood test
results obtained ureum 55 mg/dL (normal 20-40 53. A 74 years old man was admitted to the hospital
mg/dL) and creatinine 1.7 mg/dL (normal <1.1 for swollen ankle, hands and eyelids. Her skin was
mg/dL). Obtained urine lab protein 2 + and 50-60 pale, damp and cool. The patient’s vital signs
red blood cells. were: blood pressure 170/90 mmHg, heart rate
Which of the following diuretics is the most 100 beats/min, deep respiration 18 breath/min,
appropriate for treating the patient above? and temperature 37.8°C. The patient had no urine
A. Acetazolamide output and the laboratory report showed that the
B. Furosemide patient’s blood urea nitrogen [BUN], creatinine,
C. Hidrochlorothiazid potassium and phosphate level s was all higher
D. Spironolactone than normal. A chest x-ray showed white, fluffy
E. Triamteren patches that spread outward from the hilar areas
to the peripheral border of both lungs. Her left
ventricle appeared moderately enlarged. Which of
49. A 10 month pregnant woman complained lower the following is the most likely caused of an altered
abdominal pain, and suffering from dysuria for 48 renal function in this case?
hours. Patient has mild fever, urine analysis A. Elevated BUN level
showed 1+ protein, but did not find any glucose, B. No urine output
and blood. Gram stain shows a gram negative C. Heart failure
bacteria. D. Pulmonary disease
Which of the following drugs is the most E. Infection
appropriate for this patient condition?
A. Chloramphenicol 54. A 50-year-old woman went to a Puskesmas
B. Cefixime (ibu hamil), ga bleh complaining of edema in both legs which became
quinolone worse along the day. Her Blood pressure is 160/95
C. Cephadroxil mmHg, pulse rate 70/min. She also had a history
D. Cotrimoxazole of an ischemic stroke. Laboratory test are,fasting
E. Tetracycline blood sugar 150 mg/dL (N: <121 mg/dL), serum
6
creatine 2.1mg/dL (N: <1.1 mg/dL), ureum 100mg 59. On physical examination to a 60-year old male
%, urinary protein ++. (CKD) revealed hypertension and subfebril. He has had
Which of the following is the most appropriate drug history of nausea, vomit and right lateral
should be given to the patient in order to prevent abdominal pain for 2 months. Pyelogram showed
secondary stroke attack? that the right kidney smaller than of the left,
A. Propranolol asymmetrically contracted with deformity of the
B. Captopril calyceal system. The right nephrectomy was
C. Reserpin performed which showed contraction and it has
D. Hydrochlorothiazide irregular granular surface. The parenchyma is
E. Furosemid atrophic and replaced by fibrosis. The microscopic
showed glomerular sclerosis, hyalinization and
55. A 63-year-old man, came to a clinic with of pain on atrophy in cortical area. Another area showed
her knees, there are also looked swelling and fibrosis, with lymphocytes aggregation.
redness at her both of knees. She said that she Which of the following lesions could likely to
also suffering edema in both legs which became happen in this kidney?
worse along the day. Her Blood pressure is A. Renal cell carcinoma
180/100 mmHg, pulse rate 90/min. Laboratory test B. Chronic pyelonephritis
result: Hb: 10.8 gram%, serum creatine 2.7mg/dL C. Acute nephritis
(N: <1.1 mg/dL), ureum 80mg%, urinary protein + D. Nephrotic syndrome
+. You are suggesting giving an aspirin (NSAID) E. Diabetic nephropathy
h3X500 mg with caution.
Which of the following is the most reason that you
give the drug with caution? 60. A 5-year-old boy child who had a palpable
A. It is ineffective abdominal mass was brought to hospital by his
B. It causes hypokalemia mother. The child later had abdominal distention
C. It worsen anemia from bowel obstruction. An USG revealed a 6cm
D. its metabolite is nephrotoxic left renal mass with necrosis and hemorrhage.
E. It decrease kidney filtration The microscopic showed embryonal tissue with
abortive of tubular and glomerular structures.
56. A 55-year-old male complained at the physician’s The depicted lesion will most likely be,
office of his hematuria for 2 weeks now. Urine A. Teratoma
cytologic examination showed malignancy cells. A B. Retinoblastoma
cystoscopic examination found a 2 cm, exophytic, C. Wilms’tumor
friable mass on the right bladder dome. D. Hanarthoma
A biopsy of this lesion will likely to show findings E. Hemangioma
that mostly consistent with,
A. Papilloma 61. A 45 years old man, came to the hospital. And told
B. Verrucous carcinoma the doctor that he had episodic severe pain in the
C. Transitional carcinoma left abdomen and radiated into his flank and
D. Squamous carcinoma sometimes into his testical (ipsilateral). From USG
E. Adenocarcinoma the doctor found 6 mm stone in his urinary tract.
57. Which of the following risk factors is most likely to What location is the most possible for the
have led to development of this lesion? obstruction?
A. Diabetes mellitus A. Upper ureter
B. Recurrent urinary tract infection B. Middle ureter
C. Therapy with methicillin C. Lower ureter
D. Cigarette smoking D. Vesicae urinariae
E. Tuberous sclerosis E. Urethra

58. A 50-year old male was present with a month-long 62. If the patient felt the pain radiated from left
history of flank pain and hematuria. An USG abdomen to the testicle cause of a stone in his
abdominal imaged a 3-cm bosselated mass which ureter, what level of sympathetic fibers is the most
protruded from cortical in the right kidney. The responsible for ureter?
cytologic examination revealed malignant cells. A. T 7-8
A biopsy of this mass will likely to show findings B. S 2-4 (parasimpatik)
that mostly consistent with, C. T 11-12 (simpatik bladder)
A. Wilms tumor D. L1-3
B. Renal Cell Carcinoma E. L4-5
C. Transitional carcinoma
D. Squamous carcinoma 63. A 50-year-old man came to his doctor and told the
E. Undifferentited carcinoma doctor that for the last 2 months, he felt burning on
urination and the color was dark. The doctor told
him to check his urine. And the result:
7
pH :7 (N : 4.5 – 8) , WBC: 15-20/HPF (N : 0- What is the most likely glomerular lesion in this
2/HPF) ; RBC : 5-10/HPF (N: 0/HPF ). Urine patient?
sediments show crystals with cuboid shape. (ca A. Membranoproliferative
oksalat) glomerulonephritis
What is the most likely metabolic abnormality? B. Minimal change disease
A. Hyperoxaluria C. IgA nephropathy
B. Hypercalciuria D. Diabetic nephropathy
C. Hypercitraturia E. Focal segmental glomerulosclerosis
D. Hyperuricuria
E. Renal tubular acidosis 68. A 4-year-old-boy was brought to hospital with
puffiness around the eyes, especially in the
64. A 42-year-old man came to the hospital and said morning and swelling over the legs. His mother
that his urine’s color is red and he saw vermiform also complaint he has foamy urine. Urine sample
clot. From his statement, could you tell the shows protein ++++ (>3.5 per 1.73 m 2 per 24
possible source of his problem? hour), albumin level 1.5g/dL (normal=3.5-5g/dL).
A. Ureter What is the most likely underlying mechanism in
B. Urethra this patient?
C. Bladder What is the principal management in this patient?
D. Prostate A. Albumin 0,5-1 g/kg, iv, follow with
furosemide 1-2 mg/kgBB/iv
65. A 50-year-old man is hospitalized for acute B. Prednisone 40 mg/m2/day
myocardial infarction. He has decreased cardiac C. Prednisone 60 mg/m2/day
output with hypotension requiring multiple pressor D. Reduce sodium intake
agents. His urine output drops over the next 3 E. Alkylating agent
days. His serum urea nitrogen increases to 59
mg/dL, with creatinine of 2.9 mg/dL. Urinalysis 69. A 5 day old baby was hospitalized due to indirect
reveals no protein or glucose, a trace blood, and hyperbilirubinemia. On the third day hospitalization
numerous hyaline casts. Five days later, he he had fever and then he was given amoxicillin
develops polyuria and his serum urea nitrogen (beta lactam yang bikin AIN) and gentamycin
declines. intravenously. Three days later his urine output
Which of the following pathologic findings in his diminished and the laboratory test revealed urea
kidneys is most likely to have caused his nitrogen 25 (3-12) mg/dL, creatinine 0.9 (0.2-0.4)
azotemia? mg/dL.
A. Patchy tubular necrosis What is the most likely diagnosis in this patient?
B. Fusion of podocyte foot processes A. Urinary tract infection
C. Glomerular crescents B. Sepsis
D. Hyperplastic arteriolosclerosis C. Acute renal failure
E. Mesangial immune complex D. Chronic renal failure
deposition E. Acute tubular nephritis

66. A 26-year-old diabetic woman is seen in the ER for A 3 year old boy was brought to emergency unit
sore throat. Rapid strep test is positive for Karawaci Hospital with severe diarrhea since three
streptococcal pharyngitis and she was started on days ago. On arrival he is not fully alert. His vital sign
ampicillin 500 mg four times a day. Three days are as follows: blood pressure is 80/60 mmHg, pulse is
later, she develops hematuria associated with a not palpable, respiratory rate was 36 breaths per
low grade fever. On physical examination, she has minute, and body temperature was 38.4oC. On
a maculopapular rash and a temperature of 38 C. physical exam there is sunken anterior fontanel, dry
Laboratory studies show: serum creatinine 3.6 mucous membranes, sunken eyes, lack of tears, poor
mg/dl, WBC 8,700 with 56% PMN, 25% lymphs, skin turgor, and the capillary refill more then 2
3% monos and 15% eosinophils. Urinalysis: pH seconds. The daily urine volume is less than 400 mL.
6.2, protein 2+, blood 3+, 65 RBCs/HPF, 20-30 70. In this patient, the tubular epithelial cells are
WBCs/HPF, 3-4 WBC casts/HPF. ASTO value is particular susceptible to ischaemic injury due to:
increased. A. Little charged surface for reabsorption
What is the most likely diagnosis? B. Low metabolic rate and oxygen
A. Diabetic nephropathy consumption
B. IgA nephropathy C. Decreased levels of salt reaching the
C. Acute interstitial nephritis distal tubules
D. Acute pyelonephritis D. Loss of polarity that cause
redistribution of membrane proteins
67. A 40-year old man with arthritis has been using E. Passive transport systems for ions
ibuprofen (nonsteroidal anti-inflammatory) every 6 and organic acids
hours. He developed renal failure associated with
nephrotic range proteinuria.
8
71. The laboratory test in this patient reveal 76. A 64-year-old male presents with hematuria.
haemoglobin 15 g/dL( 11.5-15.5 g/dL), hematocrit Examination shows a flank mass and he has
46% (35-45%), WBC 7,000/mm3 (5,500- elevated hemoglobin. Pathology of the surgical
15,500/mm3), platelets count 260,000/mm3 specimen shows a clear cell renal cell carcinoma.
(150,000-400,000 mm3), urea nitrogen 40 mg/dL What is the cell of origin of this tumor?
(5-18 mg/dL), and creatinine 1.7 mg/dL (0.3-0.7 A. Glomerulocyte
mg/dL). B. Glomerular endothelium
The most likely diagnosis in this patient is: C. Glomerular epithelium
A. Acute tubular nephritis D. Proximal tubular epithelium (clear ell n
B. Prerenal acute kidney injury papilla)
C. Intrinsic acute kidney injury E. Histiocyte
D. Postrenal acute kidney injury
E. Chonic renal failure Distal (klomoford, oncodi)

A 10 year old girl was brought to your clinic with disuria 77. A 32-year-old female presents with headaches.
since 5 days ago. On physical exam she is healthy Investigation reveals hypertension (180/110
with no significant findings. From urine culture is found mmHg) and very high level of aldosterone
Eschericia coli with CFU ≥ 100,000. presumed due to a tumor. What of the following is
72. What is the most likely diagnosis in this patient? characteristic of this condition?
A. Acute pyelonephritic A. Elevated serum sodium levels
B. Reflux nephropathy B. Elevated serum potassium levels
C. Uncomplicated urinary tract infection C. Elevated urinary cortisols
D. Complicated urinary tract infection D. Elevated urinary bicarbonate
E. Chronic urinary tract infection
78. An 18-year-old male presents to emergency
73. Factors unfavorable to bacterial growth in this following a motorcycle accident. There is
patient’s urine is: significant blood loss. In the ER, his BP is 90/60
A. A low pH mmHg and his pulse is 120 x/min. What
B. A low concentration of urea physiological response occurs in the kidney?
C. The absence of organic acids from a A. Decreased urine osmolality
diet B. Reduced release and action of ADH
D. A low osmolality C. Beta adrenergic stimulation of the
E. The presence of normal vasculature
microorganism D. Vasoconstriction of the renal arterioles
E. Reduced aldosterone production and
74. An 8 years old boy presents with decreased urine sodium loss
output, hematuria, high blood pressure, proteinuria
add rising creatinine. One week ago he had a 1. Mr. Edi, 42 year old male, comes to the Siloam
fever and sore throat. A renal biopsy is carried out hospital with muscular weakness, mild headache
reveals diffuse proliferative glomerulonephritis. after eating salted fish. Previously he never eats
Electron microscope will show abnormal deposits salted fish. Usually he drink only small amount of
in what location? water. He said that usually his blood pressure is
A. Between basement membrane and 100/70 mmHg. His blood pressure 140/90 mmHg.
endothelial cells of the glomeruli The doctor sends him to the laboratory. The result
B. Between the basement membrane shows serum sodium is 150 mEq/L, potassium is
and epithelial cells of the proximal 3.5 mEq/L, glucose 150 mg/dl.
tubule In this condition, what will the kidney do?
C. Between the basement membrane A. Increase GFR
and epithelial cells of the glomeruli B. Increase obligatory water reabsorption
D. Within the juxtaglomerular apparatus C. Increase facultative water reabsortion
D. Increase renin secretion
75. A 45 years old female has a long history of poorly E. Increase aldosteron secretion
controlled diabetes. She presents now with
edema, hypertension and massive proteinuria, 2. To cover the above function, what mechanism is
hematuria and hyperlipidemia. Renal biopsy shows used?
hyaline arteriosclerosis of the arterioles. What A. Increase cardiac output
changes will be present in the glomeruli? B. Increase frequency of the heart
A. Diffuse proliferative glomerulonephritis C. Increase secretion of renin
B. Membranous proliferative D. Increase secretion of antidiuretic hormone
gomerulonephritis E. Increase secretion of aldosteron
C. Membranous glomerulosclerosis .
D. Focal segmental glomerulosclerosis 3. Which factor support the above mechanism?
E. Nodular glomerulosclerosis A. Filtration rate of the glomeruli
9
B. Active transport of the tubular cell Which of the following is the most likely
C. High osmolality of the interstitium of medulla appropriate for restoring the diuretic response?
D. Obligatory water reabsortion A. add with hydrochlorothiazide
E. Osmolality of the filtrate in distal convoluted B. add with metolazone
tubule C. replace with furosemide
D. replace with hydrochlorothiazide
E. increasing the dose of
4. If the urinalysis is also done, what will you see about chlorthalidone
the glucose in urine?
A. positive ++++ 9. A 75-year-old woman with hypertension is being
B. positive +++ treated with thiazide. Her blood pressure is
C. positive ++ decrease to 120/76 mm Hg. After several months
D. positive+ on the medication, she complains of being tired
E. negative and weak. Which of the following substance in
analysis of blood indicates low values?
5. In a kidney laboratory, using a micropipette, we can A. calcium
measure the hemodynamic pressure of afferent B. uric acid
arteriole, pressure in Bowman capsule, and osmotic C. potassium
pressure in glomerular lumen. If the pressure in D. sodium
afferent arteriole is 40 mmHg, in Bowman capsule 1 E. glucose
mmHg, the osmotic pressure of glomerulus is 30
mmHg, the effective filtration pressure in glomerulus is: 10. Your patient, a 45-year-old man, usually lives near
A. 71 mmHg the beach, is planning to have a vacation that
B. 69 mmHg includes a short hike to the top of Mount Everest
C. 11 mmHg (altitude approx. 29.000 feet above sea-level).
D. 9 mmHg You are concern about “mountain sickness”. He
E. – 9 mmHg has no other significant medical conditions and
takes no other drugs that would interact with the
6. A patient taking an oral diuretic for about six drug you will prescribe for his trip.
months, visits your clinic with elevated fasting and Which of the following drug you recommend?
postprandial blood glucose levels. You check the A. acetazolamide
patient’s HbA1C and find it is elevated compared B. amiloride
with normal baseline values obtained 6 months C. bumetabide
ago. You suspect the glycemic problems are D. furosemid
diuretic-induced. Which of the following drug is the E. spironolactone
most likely cause?
A. asetazolamide 11. A patient with tuberculosis develops
B. amiloride bright orange-red urine after he drank his anti-
C. chlorothiazide tuberculosis drugs and calls his physician in a
D. spironolactone panic because he is afraid he is bleeding into the
E. triamterene urine. The patient has no other urinary tract
symptoms. Which of the following medication is
7. Your patient, a 55 year-old man, with heart failure, the most likely to produce this side effect?
unacceptably low cardiac output, and intense A. Ethambutol
reflex-mediated sympathetic activation of the B. Isoniazid
peripheral vasculature that is attempting to keep C. Pyridoxine
vital organ perfusion pressure sufficiently high. The D. Rifampicin
patient is edematous, and has ascites, because of E. Streptomycin
the poor cardiac function and renal compensations
for it. Which of the following drug should be 12. A 26 year-old young man presents with the
avoided in this patient? symptoms of gonorrhea. This condition is often
A. amiloride associated with infection due to Chlamydia
B. ethacrynic acid trachomatis.
C. hydrochlorothiazide Which of the following quinolones would be the
D. mannitol best choice for treating him?
E. spironolactone A. ciprofloxacin
B. nalidixic acid
8. A 63-year-old-man, with a history of heart failure C. norfloxacin
and edema fails to respond adequately to D. levofloxacin
maximum recommended dosages of E. ofloxacin
chlorthalidone.

10
13. A jaundiced 1-year premature infant with elevated surgery. After incising the scrotum, the surgeon
free bilirubin is seen in the premature baby contemplates the approach
nursery. The mother had received antibiotic to the parenchyma of the testes.
combination for a urinary tract infection (UTI) 1 Which layer is the most near to the testicular
week before delivery. parenchyma?
Which of the following is the most likely cause of A. Fascia spermatica externa
the baby’s kernicterus ? B. Tunica albuginea
A. cefixime C. Fascia sprematica interna
B. amoxicillin D. Scarpa fascia
C. azithromycin E. Tunica dartos
D. erythromycin
E. cotrimoxazole 19. A 18-year-old man is noted to have dysuria for
several days, and the doctor diagnose that she suffers
14. A 30 year-old woman with a history of recurrent from urinary tract infection.
urinary tract infection. Since 5 days ago, she felt Which of the following structure of the urinary tract
dysuria, urgency, and frequency. Culture of urine is the most vulnerable to
sample indicated that the offending organism is get infection?
Escherichia coli. She receives ciprofloxacin and A. Kidney
the symptoms disappear. B. Ureter
Which of the following process is inhibited by C. Urinary bladder
ciprofloxacin? D. Urethra
A. cell-wall synthesis E. Adrenal gland
B. folic acid synthesis
C. protein synthesis 20. A 70 year old man came to the emergency
D. topoisomerase II department of Karawaci Hospital with
E. DNA polymerase complaint of incomplete urination since 6 months
ago. The urologist performed
15. A 30 year old man with motorcycle accident comes rectal examination and found an enlargement of
to the emergency. In the his prostate.
physical examination, the doctor found that his left Which of the following structure is affected by the
kidney has severe injured. enlargement?
And the doctor suggested operating his kidney. A. urethra pars membranacea
During the removal of a patient’s kidney, B. urethra pars spongiosa
Which of the following structure is the most anterior C. sphichter urethra interna
within the renal sinus? D. urethra pars prostatica
A. Renal arteries E. orificium urethra externa
B. Renal vein
C. Major calyx A 4-year-old-boy was brought to hospital with puffiness
D. Minor calyx around the eyes, especially in the morning and
E. Renal pelvis swelling over the legs. His mother also complaint he
has foamy urine. Urine sample shows protein ++++
16. You wish to examine the hilum of the left kidney (>3.5 per 1.73 m2 per 24 hour), albumin level 1.5g/dL
during surgery. (normal=3.5-5g/dL).
Which of the following structures must be elevated?
A. Stomach 21. What is the principal management in this patient?
B. Suprarenal gland
C. Ascending colon F. Albumin
D. Duodenum
E. Liver G. Furosemide
17. To elevate the kidney within the renal fascia and
the perirenal fat, which of the H. Prednisone
following muscles must be reflected or incised from
the fascia? I. Diet restriction
A. Diaphragm
B. Psoas J. Cyclosporin

C. Quadratus lumborum
D. Transverses abdominis
E. Iliacus 22. During 4 weeks-treatment, the
urine sample shows protein +.
18. An 18-year-old man is noted to have probable
testicular cancer. He undergoes
11
What is your conclusion in this patient? had fever and then he was given amoxicillin and
gentamicin intravenously. Three days later his urine
A. Initial responder output diminished and the laboratory test revealed
urea nitrogen 25 (3-12) mg/dL, creatinine 0.9 (0.2-0.4)
mg/dL.
B. Late responder
25. What is the most likely diagnosis in this patient?
C. Drug resistant
F. Urinary tract infection
D. Drug dependent
G. Sepsis
E. Remission
H. Acute renal failure
A 13 year-old-girl was brought to emergency unit with
I. Chronic renal failure
seizure. Her mother complaint she had fever 2 weeks
ago, but has recovered. Now she has swelling around
the eyes and legs and she has dark urine. From J. Acute tubular nephritis
physical exam, she is unconscious. Her blood
pressure is 180/120 mmHg. Urine sample shows 26. What is the most possible cause in this patient?
erythrocytes 20/hpf and protein +. Albumin level within
normal limit.
F. Ischemia
23. What other laboratory findings to confirm the
diagnosis in this patient?
G. Direct toxic injury
F. Decreased anti streptolisin O (ASO),
H. Complex immune response
increased DNase-B, decreased C3

I. Urinary obstruction
G. Increased anti streptolisin O (ASO),
decreased DNase-B, decreased C3
J. Drug hypersensitivity
H. Decreased anti streptolisin O (ASO),
increased DNase-B, increased C3

I. Increased anti streptolisin O (ASO), 27. What is the pathognomonic laboratory finding
decreased DNase-B, increased C3 (urinalysis) in this patient?

J. Increased anti streptolisin O (ASO), A. Increased osmolality


increased DNase-B, decreased C3
B. Proteinuria

C. Muddy brown casts


24. What is the most likely mechanism that underlying
this patient’s disease? D. Erythrocyte casts

A. Streptococcal bacteria infection E. Haemoglobinuria

B. An inflammation of the kidney tubules A 3 year old boy was brought to emergency unit
Karawaci Hospital with severe diarrhea since three
C. Increased permeability of glomerulus days ago. On arrival he is not fully alert. His vital sign
membrane are as follows: blood pressure is 80/60 mmHg, pulse is
not palpable, respiratory rate was 36 breaths per
minute, and body temperature was 38.4oC. On
D. Small pores in the podocytes permit
physical exam there is sunken anterior fontanel, dry
erythrocyte loss
mucous membranes, sunken eyes, lack of tears, poor
skin turgor, and the capillary refill more then 2
E. Heparan sulfate mucopolysaccharide is seconds. The daily urine volume is less than 400 mL.
low so erythrocyte cross the barrier
28. In this patient, the tubular epithelial cells are
A 5 day old baby was hospitalized due to indirect particular susceptible to ischemic injury due to:
hyperbilirubinemia. On the third day hospitalization he F. Little charged surface for reabsorption
12
G. Low metabolic rate and oxygen G. Reflux nephropathy
consumption
H. Uncomplicated urinary tract infection
H. Decreased levels of salt reaching the
distal tubules I. Complicated urinary tract infection

I. Loss of polarity that cause redistribution of J. Chronic urinary tract infection


membrane proteins

J. Passive transport systems for ions and


organic acids 32 .Which of the following factor is unfavorable to
bacterial growth in patient’s urine?

F. A low pH (5.5 or less)


29. The laboratory test in this patient reveal
haemoglobin 15 g/dL(11.5-15.5 g/dL),
G. A low concentration of urea
hematocrit 46% (35-45%), WBC
7,000/mm3(5,500-15,500/mm3), platelets count
H. The absence of organic acids from a diet
260,000/mm3 (150,000-400,000 mm3), urea
nitrogen 40 mg/dL (5-18 mg/dL), I. A low osmolality

and creatinine 1.7 mg/dL (0.3-0.7 mg/dL). J. The presence of normal microorganism

What is the most likely diagnosis in this patient?

F. Acute tubular nephritis 33. Which of the following antibiotic is the most
appropriate for this patient?
G. Prerenal acute kidney injury
A. Ceftriaxone
H. Intrinsic acute kidney injury
B. Cefotaxime
I. Postrenal acute kidney injury
C. Ceftazidime
J. Chonic renal failure
D. Cotrimoxazole

E. Metronidazole
30. What is the most prior management in this patient?

A. Bicarbonate

B. Acute Dialysis A 55-year-old man refers to ICU after the procedure


open reduction of femur fracture and bleeding intra
C. Furosemide operative. At the third day he became apatis, fever
38,5 Celcius with blood pressure 110/60 mmHg, pulse
D. Antibiotic 55 x/min, respiratory 20x/min, and urine output
decreased to 10 cc/hour in 24 hours. At laboratory
E. Fluid replacement therapy examination findings creatinine 4 mg/dL, BUN 250
mg/dL, sodium 130 mEq/L, chloride 90 mEq/L,
potassium serum 7.5 mEq/L, blood sugar 200 mg/dL.
A 10 year old girl was brought to your clinic with
dysuria since 5 days ago. On physical exam she is From blood gas analysis finding pH 7.1, pCO2 30
healthy with no significant findings. From urine culture mmHg, pO2 90 mmHg,
is found Eschericia coli wih CFU ≥ 100,000.
31. What is the most likely diagnosis in this patient?
HCO3 18 mEq/L. ECG examination finds widening of
QRS wave.
F. Acute pyelonephritic
13
(E) End State Kidney Disease

34. What is the most critical condition can cause life


threatening the patient? 38. Which of the following statements in true of
hematuria caused by bladder cancer?
(A) Nothing for awareness (A) It is usually accompanied by discomfort and
painful voiding
(B) creatinine and BUN at very high level (B) It is intermittent and painless
(C) It occurs in a minority of patients with bladder
(C) hyperkalemia cancer
(D) It commonly causes anemia
(D) metabolic acidosis (E) It occurs primarily only in the initial phase of
the urinary stream
(E) Severe hyponatremia
39. A 36-year-old
male truck driver is referred to you for evaluation
of recurrent urolithiasis. The patient has passed
35. What is the best choice of treatment for the clinical more than 20 calcium oxalate stones since age
situation for the patient? 18. Previous therapy with hydrochlorothiazide
and cellulose phosphate has been ineffective.
(A) Food restriction with potassium composition Physical examination is normal. Laboratory
finding: Serum creatinine 1.0 mg/dL, Serum uric
(B) Give K sparing diuretic acid 8.9 mg/dL, Urine pH 5.0
Radiographs of the abdomen show smooth renal
silhouettes measuring 13.6 cm on the right and
(C) Give regular Insulin 10 U in dextrose 50%
14.2 cm on the left, and no radioopaque at
intravenous
kidney region. From sonography findings
multiple hyperechoic shadow from both kidney
(D) Give bicarbonate sodium intravenous
with measure about 2 – 5 mm.
Which of the following treatment is the most
(E) Give forced diuretic
effective for this patient?
(A) Dietary sodium restriction

36. After the choice procedure, what is the next (B) Dietary calcium supplementation
treatment for the patient? (C) High fluid intake and long-term
urine acidification
(D) High fluid intake and allopurinol
(A) Waiting for the general condition of the
(E) Shock wave lithotripsy
patient became more better
40. A 40-years-patient arrives at ER department with a
(B) Give diuretic until the urine output will be
transthoracic gunshot wound and microscopic
normally
hematuria.
What is the most appropriate supporting diagnostic
(C) Hemodialysis immediately
technique?
(A) Abdominal sonography
(D) Give sodium bicarbonate until pH of (B) Intravenous pyelography
blood became normal (C) Immediate laparatomy
(D) Clinical observation on the hospital
(E) Intubation and ventilator to maintain the ward
respiratory (E) Immediate CT with i.v contrast agents

41. You are asked to evaluate a 38-year-old woman


who has a ten-year history of bipolar disorder. She
37. According to RIFLE criteria, what is clinical has taken lithium carbonate for seven years. For
condition of the patient? the past two years, she has had severe polyuria
(urine volume up to 10 L/24 hr). She refuses to
(A) Risk discontinue lithium therapy.
What is the most appropriate agent to reduce the
(B) Injury polyuria?
(A) Amiloride
(C) Failure (B) Chlorpropamide
(C) Furosemide
(D) Loss (D) Hydrochlorothiazide
14
(E) Acetazolamide parasympathetic system in controlling the
urinary bladder?
42. A man 45 years old come with bruise at lower A. Relaxes external sphincter
abdominal and pain. He is suspected with B. Relaxes internal spinchter
bladder injury, C. Relaxes detrussor muscle
Which of the following statement is true about D. Contracts detrussor muscle
bladder injuries? E. Contracts anal spinchter
(A) They are present 6% -10% in patients with
pelvic fracture 46. The parasympathetic outflow in the spinal cord
(B) More than 80% come with urine retention occurs at levels:
(C) Delayed diagnosis is the rule A. T3,4
B. T 6-10
(D) Bladder contusions commonly result in C. S1,2,3
gross hematuria D. S2,3,4
(E) They are associated with a 50% rate of E. L1,2,3
urethral tear
47. A 75 year old man had urine incontinent since 3
43. A 51-year-old man, who is hospitalized for months ago. He had been diagnosed dementia by
treatment of Pneumocystis pneumonia, is found to the doctor since 6 months ago. On examination he
be HIV seropositive. There is no family history of had no prostate problems. What is the cause of his
kidney disease or hypertension. Temperature is urination’s problem?
38.1 C (100.6 F). Pulse rate is 102 per minute, and F. Impaired frontal inhibition
respirations are 23 per minute. Urinalysis show G. Impaired pontine inhibition
Protein 3+; 0–2 RBCs; 0–1 WBC/hpf, and Urine H. Sympathetic system dysfunction
protein is 6230 mg/24 hr. Ultrasonography I. Parasympathetic system dysfunction
demonstrate large, echogenic kidneys. J. Sensoric system dysfunction
What pathology feature is demonstrated in the
Kidney biopsy? 48. A 40 years old woman, body weight 50 kg, came to
(A) Postinfectious glomerulonephritis emergency unit with diarrhea (frequency 8
(B) Membranoproliferative times/day) since 2 days ago. Her urine volume
glomerulonephritis was 150 cc/12hours. She was conscious, BP
(C) Membranous glomerulopathy 85/60 mmHg, pulse 105 x/min, RR 20 x/min,
(D) Collapsing glomerulopathy temperature 36,5 C. Laboratory results: Hb 12 gr
(E) Minimal change disease %, Ht 40%, WBC 9000, Platelet 263.000, ureum
104, creatinine 3,7.
44. A 36-year-old woman comes to the emergency What is the most appropriate initial treatment for
department because of severe, sharp right flank the patient?
pain of four hours' duration. During physical A. Give an antibiotic
examination, she cannot lie because of pain. B. Ringer lactate infusion
Temperature is 39.0 C (102.2 F), and pulse rate is C. Give vasopressin
120 per minute. The right flank is tender to deep D. Glucose infusion
palpation. Leukocyte count is 18,000/cu mm. E. Oral rehydration
Serum creatinine is 2.0 mg/dL. Urinalysis shows
protein 1+; 10–20 WBCs, 10–20 RBCs/hpf. 49. A 44 year old man was hospitalized for systemic
Radiograph of the kidneys, ureters, and bladder fungi infection. He was on Amfoterisin B treatmet
shows a 1.5-cm calcific density in the area of the for 7 days. On examination, he conscious, BP
right mid-ureter. Both kidney silhouettes measure 120/80 mmHg, pulse 80 x/min, RR 16 x/min,
approximately 12 cm pole-to-pole. temperature 36,7 C. Lab: Hb 13 gr%, HT 37%,
Ultrasonography shows a hydronephrotic right WBC 8600, Platelet 340.000. Urinalysis: RBC 0-
kidney and contracted left kidney. 1/HPF, WBC 2-5/HPF, protein (-), granular cast (+).
What is the most appropriate management for this Ureum 91, creatinin 2, 0. Volume urine 800 cc/24
woman? jam
(A) Intravenous urography What is the most likely diagnosis?
(B) Intravenous ketorolac to relax the ureter A. Acute Tubular
(C) Intravenous fluids that allow the stone to necrosis
pass B. Acute
(D) Immediate percutaneous nephrostomy Glomerular Nefritis
(E) Emergency lithotripsy C. Acute
Interstitial nephritis
45. Normal micturition requires that the urinary D. Acute Renal
bladder and the urethral sphincter work together Failure
as a coordinated unit to store and empty urine. E. Chronic Renal
Which of the following is the effect of failure
15
in low extremity. Lab exams: Hb 13,8 g/dL, WBC
50. A young woman went to a physician because 10.950 mL, Uric acid 7,5 mg/dL, Ureum 86 mg/dL,
pain during urination and frequency. On Creatinin 1,4 mg/dL.
examination, she had has suprapubic pain, but
no fever. Her urine was cloudy and shows 54. What is the most appropriate nutrition for this
microscopic evidence of erythrocytes, pyuria, patient:
and gram-positive bacteria. In the past medical
history, she never had any symptoms like this A. Restrict water to 1000 ml/day.
one. Which organism is most likely responsible B. No restriction for food consistency.
for this patient’s illness? C. Drink fruit juice as much as possible.
F. Klebsiella pneumonia D. Patients should drink as much as possible.
G. Pseudomonas aeruginosa E. Ask him to drink while thirst only. Choose dry
H. Escherichia coli foods.
I. Proteus mirabilis
J. Staphylococcus saprophyticus 55. Protein recommendation:
A. No need to restrict protein
51. Which urine specimen is most appropriate for B. Use meat based protein not from the
the culture examination for the patient above? legumes or beans
A. Suprapubic C. Limit protein high in uric Acid such as
B. Clean catch beans, seeds
C. Straight catheter D. Limit protein 0,7 gram/kg BB
D. Indwelling catheter
E. 24 hours urine collection After a year he comes with weakness, nausea, muscle
cramp, and itchy. Physical exam: BW 52 kg, edema in
52. A pregnant woman was complaining fever, pain both legs and palpebras. Blood pressure 170/100
when urinating, blood in her urine, and severe mmHg. Lab exam: Hb 7,6 g/dL, Ureum 196 mg/dL.
lower back pain. The physician submits her urine Creatinine 3,7 mg/dL. Uric Acid 11,8 mg/dL. Organic
for culture; the next day, the laboratory reports Phosphorus 11 mg/dL, Calcium 3,4 mg/dL Sodium 147
an organisms growth count of greater than mmol/L, Potassium 5,6 mmol/L.
105/ml of urine. The organism grew well on the
blood agar and producing β–hemolytic colonies. 56. What is the most appropriate method to nourish
Which important virulence factor for the him:
organism is most likely responsible for this A. Parenteral nutrition and nil by
patient’s illness? mouth
A. Capsule B. Give a fruit juice and add sugar.
B. P-fimbriae C. Give milk in enough calories.
C. Endotoxin D. Temporary no fruits and give
D. Phospolipase C blended vegetable.
E. Lipooligopolysaccharide
57. About calcium phosphorus balance in this patient:
53. A male patient complained of a sharp pain in his
suprapubic radiating down to his perineum, A. He needs milk fortified calcium.
accompanied by hematuria. He reported that he B. Restrict protein intake
had a few days of urinary urgency, frequency, C. Reduce meat base protein and
and burning when urinating. On examination, his nuts.
temperature was 37,5ºC; other vital sign were D. Reduce protein and give high
normal; and he appear to be in pain. A urinalysis calories.
shows erythrocytes, leukocytes esterase and a
high pH.
Gram stain demonstrates negative coccobacili. 58. A 55 year old male complains hematuria at
Which antigenic composition is most likely thephysician’s office for 2 weeks.
responsible for this illness? A cystoscopic examinations showed a 2
A. Hemolysin cm,exophytic,friable mass on the right bladder
B. Catalase dome.
C. Fibronectin A biopsy of this lesion show findings most
D. Opa protein consistent with a/an
E. Cord factor F. Papilloma
G. Verrucous carcinoma
A 45 years old man comes to Siloam Hospitals with H. Transitional carcinoma
low back pain and painful sensation when urinating. I. Squamous carcinoma
Anamnesis: he has a history of kidney stone. Physical J. Adenocarcinoma
examination: pallor, fluctuating pain, blood pressure
140/90 mm Hg. Weight 54 kg, Height 167 cm. edema
16
59. A 60 year old male has had multiple recurrent costovertebral angle tenderness. Renal
urinary tract infections with 6 month history of ultrasound shows marked left hydronephrosis.
dysuria. Rectal toucher by his physician palpates Serum creatinine and CBC are normal.
prostate enlargement. Which of the following diagnostic test is the most
What is the most appropiate clinical action to relevant?
diagnose carcinoma prostate ? A. Renal biopsy
A. Trans rectal biopsy B. Renal arteriogram
B. Trans urethral biopsy C. Palpation for inguinal lymph nodes
C. Transrectal FNAB D. Antinuclear antibody test
D. Rectal toucher E. IVP
E. Open surgery
64. A 34-year-old man comes with 3-day history of
60. A 50 year old male presents with a month long low back pain, urinary hesitancy, and pain with
history of flank pain and hematuria.An USG urination. He has not had any urethral discharge
abdominal results a 3 cm bosselated mass, or recent sexual contacts. His temperature is
protrudes from cortical are in the right 38.2C, and blood pressure is 120/70 mm Hg.
kidney.Cytologic examination reveals malignant Pulse 96x/min, RR 20x/min. There is no
cells. What is most likely biopsy finding for this costovertebral angle tenderness. Abdominal
patient? examination shows no tenderness or masses.
F. Wilms tumor Rectal examination shows no tender prostate
G. Renal Cell Carcinoma and no masses.
H. Transitional carcinoma Specific gravity 1.030; Blood negative; Glucose
I. Squamous carcinoma negative; Ketones negative;
J. Undifferentited carcinoma Leukocyte esterase positive
What is the most likely diagnosis?
61. A 60 year old male experiences the onzet of A) Benign prostatic hypertrophy
headache, naussea ,vomitting fo 1 B) Cystitis
month.Physical examination reveals C) Prostatitis
hypertention, subfebril. Pyelogram showed the D) Pyelonephritis
affected kidney assymmetrically contracted with E) Urolithiasis
deformity of the calyceal system. Gross of the
affected kidney showed contracted and has 65. A 63 year old female patient came with complain
irregular granular surface.The parenchyma is of pain while urinate. She has history of
atrophic and replace by fibrosis. Microscopic recurrent urinary infections. Physical
showed glomerular sclerosis, hyalinization and examination revealed tenderness in CVA region.
atrophi in cortical area. Another areas showed Urinalysis was done and radiology finding
fibrosis, chronic inflammatory cells with reveals large, bilateral "stag horn" calculi.
limpocytes agregation. What kind of stone is likely to be demonstrated
What is the most likely lesions occur in this in this case?
kidney?
A. Calcium oxalate stones
F. Renal cell carcinoma B. Struvite stones
G. Chronic pyelonephritis C. Calcium phosphate stones
H. Acute nephritis D. Uric acid stones
I. Nephrotic syndrome E. Cystine stones
J. Diabetic nephropathy

62. A 5 year old boy child who had a palpable 66. A 55-year-old man is brought to the emergency
abdominal mass brought to hospital by his due to acute onset of severe right-sided flank
mother . The child later had abdominal distention pain. He has a 10-year history of gout. His
from bowel obstruction. An USG reveals a 6 cm blood pressure is 110/80 mm Hg, pulse is
left renal mass with necrosis and hemorrhage. 78x/min, and RR 16x/min. Examination shows
Microscopic showed embryonal tissue with normal bowel sounds and no abdominal
abortive of tubular and glomerular structures. tenderness or masses. Urinalysis shows 40
What is the diagnosis ot this patient? erythrocytes/hpf. Intravenous pyelography
F. Teratoma confirms a right ureteral calculus.
G. Retinoblastoma Which of the following underlying mechanism is
H. Wilms’tumor responsible for this patient's condition?
I. Hanarthoma A. An increase in urinary pH
J. Hemangioma B. Damage to the epithelial lining of the
ureters
63. A 62-year-old woman presents with hematuria C. Lack of inhibitors of crystal formation
and left flank pain. Physical examination reveals D. Presence of urease-splitting bacteria
17
E. Urinary supersaturation with uric acid d. Normal nephrogram

72. What is the most likely the appearance of renal


A 66 years old, came to Emergency Department with artery rupture?
shortness of breath as his chief complaint. He also got a. The kidney shows non function
nausea, vomiting and loss of appetite since 3 days b. Ekstravasation
ago. No diarrhea was occurred prior to the symptoms. c. Hydronephrosis
He was taking un regularly anti hypertension drugs for d. Normal nephrogram
20 years. Physical exam: weight 70 kg, blood pressure
190/110, Pulse 120 times/minute, and Respiratory rate 73. A 50 years old man comes to the clinic with
46 times/minute. Pale conjunctiva, fine crackles +/+ complain of abdominal colic. The result of IVP
were heard on the both side of the lung. Heart, liver demonstrates ureteric dilatation.What is the most
and spleen were normal. likely the problem of ureteric dilatation?
Lab results: Hb 7.2 g/dl, WBC 7000/mm3, trombosit a. Stone in the pelvicureter junction
225000/mm3, random glucose 135 mg/dl, Ureum 150 b. Kidney cyst
mg/dl, Creatinin 5.2 mg/dl. Proteinuria +3 c. Stone in the vesicoureter junction
d. Renal cell Ca stage I
67. What is the most possible causes of anemia
based on the case above? 74. A 65 years old man is diagnosed with Prostates
A. B6 deficiency hypertrophy. What is the most likely feature appeared
B. Erithropoeitin deficiency in cysptography?
C. B12 deficiency a. Indentation of inferior aspect of bladder
D. Iron deficiency b. Indentation of superior aspect of bladder
E. Folic deficiency c. Indentation of posterior aspect of bladder
d. Additional shadow of superior aspect of
68. What is the most possible electrolyte level show? bladder
A. Phosphate decreased
B. Calcium increased 75 A 45 years old woman is diagnosed with Bladder
C. Potassium decreased stone. What is the most likely feature appeared in
D. Phosphate increased cysptography?
E. Normal Calcium a. Indentation of inferior aspect of bladder
b. Indentation of superior aspect of bladder
69. Mrs Jane 35 years old, came to Internal c. Filling defect in the bladder
Department with complain, fatigue and d. Additional shadow of superior aspect of bladder
headache about 1 month . In physical examination:
pale, butterfly rash at her face, another
examination in normal limit. The physician
suspects that Jane suffers SLE. 1. 109 284. The consumption of oxygen by the
What’s the most important lab examination to kidney
support the diagnosis: a. Decreases as blood flow increases
a) Complete Blood Count b. Is regulated by erythropoietin
b) Complement fixation test c. Remains constant as blood flow
c) Anti Nuclear Antibody increases
d) Renal function test d. Directly reflects the level of sodium
e) Liver function test transport
e. Is greatest in the medulla
70. According to case no 1, beside protein uria what is 2. 110 285. The anion gap will increase with an
the most urinalysis result in increase in the plasma concen-tration of
this case : a. Sodium
a) Glucosuria b. Potassium
b) Leukocyte uria c. Chloride
c) Hematuria d. Bicarbonate
d) Bilirubinuria e. Lactate
e) Crystaluria 3. 111 288. In the presence of ADH, the filtrate will
be isotonic to plasma in the
71. A 25 years old man has been crashed by a truck. a. Descending limb of the loop of Henle
The doctor suspects that his kidney is rupture, then he b. Ascending limb of the loop of Henle
sent the patient to have IVP. What is the most likely the c. Cortical collecting tubule
appearance of renal parenchymal rupture in IVP? d. Medullary collecting tubule
a. The kidney shows non function e. Renal pelvis
b. Ekstravasation 4. 112 289. Sodium reabsorption from the distal
c. Hydronephrosis tubule will be increased if there

18
is an increase in concentration of 12 mg/mL and a renal vein
a. Plasma potassium concentration concentration of 9 mg/mL. Calculate the
b. Plasma volume filtration fraction (GFR/RPF).
c. Mean arterial pressure a. 0.05
d. Urine flow rate b. 0.15
e. Plasma osmolality c. 0.25
5. 113 290. ADH will be released from the posterior d. 0.35
pituitary when there is a decrease in e. 0.45
a.Plasma Na+concentration 13. 122. Use the following laboratory data to
b.Plasma volume determine the GFR.
c.Plasma K+concentration Urine creatinine concentration =196 mg/mL
d.Plasma pH Plasma creatinine concentration=1.4 mg/mL
e.Plasma Ca2+concentration Urine flow =1 mL/min
6. 114 291. If 600 mL of water is ingested rapidly, The creatinine clearance is approximately
plasma volume will increase by approximately a. 75 mL/min
a400 mL b. 98 mL/min
b200 mL c. 125 mL/min
c100 mL d. 140 mL/min
d50 mL e. 196 mL/min
e25 mL 14. 123. The electrically neutral active transport of
7. 116 293. Renin secretion by the kidney is sodium from the lumen of the kidney occurs in
increased by the
a. Increasing mean blood pressure a. Proximal tubule
b. Increasing glomerular filtration rate b. Descending limb of the loop of Henle
c. Increasing sympathetic nerve activity c. Ascending limb of the loop of Henle
d. Increasing angiotensin II synthesis d. Cortical collecting duct
e. Increasing atrial natriuretic hormone e. Medullary collecting duct
secretion 15. In metabolic acidosis caused by diabetic
8. 117 294. Na+is reabsorbed from the basolateral ketoacidosis, which of the following would be
surface of the renal epithelial greater than normal?
a. cells by a. Concentration of plasma HCO3−
b. a Na/H exchange b. Anion gap
c. Na-glucose cotransport c. Arterial PCO2
d. Na-K pump d. Plasma pH
e. Facilitated diffusion e. Blood volume
f. Solvent drag 16. 125. Decreasing the resistance of the afferent
9. 118 295. Which of the following is most likely to arteriole in the glomerulus of the kidney will
cause an increase in the glomerular filtration decrease
rate? a. The renal plasma flow
a. Contraction of mesangial cells b. The filtration fraction
b. Blockage of the ureter c. The oncotic pressure of the peritubular
c. Release of renin from the capillary blood
juxtaglomerular apparatus d. The glomerular filtration rate
d. Dilation of the afferent arterioles e. None of the above
e. Volume depletion 17. 126. If GFR increases, proximal tubular
10. 119 296. The daily production of hydrogen ion reabsorption of salt and water will increase by
from CO2 is primarily buffered by a process called glomerulotubular balance.
a. Extracellular bicarbonate Contributions to this process include
b. Red blood cell bicarbonate a. An increase in peritubular capillary
c. Red blood cell hemoglobin hydrostatic pressure
d. Plasma proteins b. A decrease in peritubular sodium
e. Plasma phosphate concentration
11. 120. Glomerular filtration rate would be c. An increase in peritubular oncotic
decreased by pressure
a. Constriction of the efferent arteriole d. An increase in proximal tubular flow
b. An increase in afferent arteriolar e. An increase in peritubular capillary
pressure flow
c. Compression of the renal capsule 18. 127. Renin release from the juxtaglomerular
d. A decrease in the concentration of apparatus is inhibited by
plasma protein a. Beta-adrenergic agonists
e. An increase in renal blood flow b. Prostaglandins
12. 121. A freely filterable substance that is neither c. Aldosterone
reabsorbed nor secreted has a renal artery d. Stimulation of the macula densa
19
e. Increased pressure within the afferent 25. 134 . Which one of the following will be
arterioles increased in a patient suffering from persistent
diarrhea?
19. 128. Patients with renal insufficiency develop a. The filtered load of HCO3−
very high plasma concentrations of urea b. The production of ammonia by the
(uremia) because of proximal tubule
a. An increased synthesis of urea by the c. H+secretion by the distal nephron
liver d. The anion gap
b. An increased reabsorption of urea by e. The production of new bicarbonate by
the proximal tubules the distal nephron
c. A decreased secretion of urea by the 26. 135 . Diuretics, such as acetazolamide, which
distal tubules produce their effect by
d. A decreased glomerular filtration rate a. inhibiting carbonic anhydrase, inhibit
e. An increased renal blood flow the reabsorption of sodium in
20. 129 . Which one of the following statements b. The proximal tubule
about aldosterone is correct? c. The thick ascending limb of Henle’s
a. It produces its effect by activating loop
cAMP d. The distal convoluted tubule
b. It produces its effect by increasing e. The cortical collecting duct
distal tubular permeability to sodium 27. 237. Which is a common finding in acute
c. It causes an increased reabsorption of glomerulonephritis?
hydrogen ion a. Pulmonary congestion due to volume
d. It has its main effect on the proximal expansion
tubule b. Hypovolemia due to tubular
e. It is secreted in response to an dysfunction
increase in blood pressure c. Uniformly progresses to chronic renal
21. 130 . The effect of antidiuretic hormone (ADH) failure if untreated
on the kidney is to d. Urine showing leukocytes and
a. Increase the permeability of the distal eosinophils
nephron to water 28. 238. Which finding is fairly specific for chronic
b. Increase the glomerular filtration rate renal failure?
c. Increase the excretion of Na+ a. Anemia
d. Increase the excretion of water b. Hyaline casts
e. Increase the diameter of the renal c. Broad casts in urinalysis
artery d. Proteinuria
22. 131 . The glomerular filtration rate will increase e. Hypocalcemia
if 29. 239. Nephrotic syndrome is associated with
a. Sympathetic nerve activity to the a. Excessive renal salt and water loss
kidney increases b. Hyperlipidemia due to lipoprotein
b. The afferent arteriolar resistance excess
increases c. Bleeding due to loss of clotting factors
c. The efferent arteriolar resistance d. Hypothyroidism due to loss of thyroid-
decreases binding globulin
d. The plasma protein concentration e. e. The outer medullary collecting duct
decreases 30. 240. A patient with chronic renal failure will be
e. Urine flow through the urethra is expected to have which of
blocked a. the following findings due to the
23. 132 . Potassium-sparing diuretics inhibit mechanisms described?
Na+reabsorption in the b. Hypercalcemic due to elevated PTH
a. Proximal tubule hormone
b. Thin descending limb of Henle’s loop c. Prolonged bleeding due to decreased
c. Thick descending limb of Henle’s loop synthesis of clotting factors
d. Distal convoluted tubule d. Anemia due to increased red cell
e. Cortical collecting duct destruction
24. 133. Which one of the following values will be e. Hypermagnesemia due to decreased
above normal in a diabetic renal excretion
a. patient with a blood glucose
concentration of 600 meq/L? 31. 241. A high fractional excretion of sodium is
b. Urine flow typically found in
c. Intracellular volume a. Heart failure
d. Plasma sodium concentration b. Urinary tract obstruction
e. Arterial pH c. Acute tubular necrosis
f. Alveolar PCO2 d. Acute glomerulonephritis
20
e. Hepatorenal syndrome bicarbonate in the urine
32. 242. Which of the following nephron segment c. Increased potassium loss through
is correctly paired with its function? extrarenal mechanisms
a. Distal tubule and bicarbonate d. Decreased fractional excretion of
reclamation water due to ADH resistance
b. Loop of Henle and potassium 38. 248. Which of the following serologic finding is
regulation associated with linear staining of the
c. Proximal tubule and urinary glomerulus on immunofluorescence?
concentration a. Anti-GBM antibody
d. Collecting tubule and water regulation b. Low complement immune complex
33. 243. Which of the following statements is true glomerulonephritis
in the management of acute renal failure? c. ANCA associated renal disease
a. Metabolic acidosis is fully corrected d. Membranoproliferative
with bicarbonate glomerulonephritis
b. Hyperphosphatemia is primarily 39. 249. Antineutrophil cytoplasmic antibody
managed with dialysis (ANCA) is typically present in which systemic
c. Low-dose dopamine is used to disease?
shorten the duration of renal failure a. Goodpasture’s syndrome
d. Hypervolemia is managed with high- b. Wegener’s granulomatosis
dose loop diuretics c. Systemic lupus erythematosus
e. Hyponatremia is corrected by d. Thrombotic thrombocytopenic purpura
administration of sodium salts 40. 252. Prerenal azotemia is associated with
34. 244. Which of the following describes bone a. High fractional excretion of sodium
abnormalities in patients with chronic renal b. Granular casts in the urine
failure? c. Use of angiotensin-converting enzyme
a. Osteitis fibrosis cystica is a result of (ACE) inhibitors in unilateral renal
oversuppression of PTH d. artery stenosis
b. Adynamic bone disease is associated e. Evolution to acute tubular necrosis if
with myopathy untreated
c. Osteomalacia is due to excessive 41. 254. Leukocytes and white cell casts in the
accumulation of magnesium urine are typically seen in
d. Hyperparathyroidism responds well to a. Radiocontrast nephropathy
1,25 dihydroxyvitamin D b. Methicillin-induced renal insufficiency
e. Amyloidosis is similar in etiology to c. Aminoglycoside nephrotoxicity
patients who are not on dialysis d. Rhabdomyolysis
35. 245. Which one of the following statements is 42. 256. Which may cause acute renal failure in
true concerning hematologic disorders in patients with nephrotic syndrome?
CRF? a. Dietary protein restriction
a. Resistance to erythropoietin is most b. ACE inhibitors
commonly due to aluminum overload c. Lipid-lowering agents
b. Erythropoietin administration is d. Loop diuretics
associated with worsening 43. 264. A 25-year-old man with flank pain is found
hypertension to have three cysts in each kidney, normal
c. The major cause of death in CRF is hepatic and renal function, and family history
sepsis is not clear. He is most likely to have
d. Abnormal bleeding responds best to a. Autosomal dominant polycystic kidney
platelet transfusion disease
e. Leukocyte function is generally b. Autosomal recessive polycystic kidney
unimpaired disease
36. 246. Which of the following measures has not c. Acquired cystic disease
been shown to retard progression of renal d. Medullary sponge kidney
failure? 44. 266. Which is an accurate statement
a. Aggressive BP control concerning diabetic nephropathy?
b. Decrease in protein intake a. Most patients with type 2 diabetes will
c. ACE inhibitors above other develop this problem
antihypertensives b. It is almost always associated with
d. Erythropoietin for anemia retinopathy in type 1 diabetes
37. 247. In patients with chronic renal failure, c. ACE inhibition is only indicated for
which of the following adaptations are normal? patients with hypertension
a. Fractional excretion of sodium d. Routine dipstick urine should be
increases due to suppression of performed to screen for early disease
aldosterone 45. 267. Which of the following is a secondary
b. Metabolic acidosis due to loss of cause for focal segmental sclerosis?
21
a. Hodgkin’s disease amount of proteinuria and “grape clusters” are
b. Colon cancer seen under light microscopy. Which of the
c. HIV disease following is the most likely diagnosis?
d. Hepatitis C infection a. Glomerulonephritis
46. 268. A patient with Crohn’s disease passes a b. Rhabdomyolysis
kidney stone; the most likely c. Nephrotic syndrome
a. composition is d. Acute interstitial nephritis
b. Calcium phosphate e. Acute tubular necrosis
c. Uric acid
d. Struvite 53. 6-31. A 50-year-old woman com- plains of
e. Calcium oxalate leakage of urine when she laughs, coughs, or
47. 277. Hyperkalemia may be caused by sneezes. After stress incontinence, the most
a. Trimethoprim common causes of this urinary leakage is
b. Albuterol a. Detrusor dyssynergia
c. Licorice b. Unstable bladder
d. Cisplatin c. Unstable urethra
d. Urethral diverticulum
48. 278. A middle-aged patient with an elevated e. Overflow incontinence
serum creatinine, hypertension, and mild
anemia comes to you for evaluation. Urine
dipstick shows trace protein without red cells 1. To elevate the kidney within the renal fascia
or cellular casts. A 24-h urine collection and the perirenal fat, the renal fascia must be
reveals 5 g of protein. The most likely etiology reflected or incised from the fascia of the?
is
a. Focal segmental sclerosis A. diaphragm
b. Hypertensive nephrosclerosis B. psoas m.
c. Amyloidosis C. quadratus lumborum m.
d. Multiple myeloma D. transverse abdominis m.
49. 279. Which of the following is a common E. iliacus m.
cause of isolated hematuria with isomorphic
red cells in the urine? 2. Young man, vomit to point where he become
a. Alport’s syndrome (hereditary hypovolemic; as evidence by an
nephritis) accompanying decrease in BP and feeling of
b. Thin basement membrane disease light-headness. The kidney respond by
c. Idiopathic hypercalciuria reducing urinary volume flow, thus limiting the
d. IgA nephropathy potential effect of hypovolemia. increase in the
50. 280. A 26-year-old woman with a history of plasma level of which of the following
mitral valve prolapse comes in with 1 week of hormones will bring about the most dramatic
fever that started 3 days after a dental decrease in urinary volume flow rate?
procedure. Her urine contains red cells and
her rheumatoid factor is elevated. Which of the A. angiotensin II
following serologic abnormalities is expected B. atrial natriuretic peptide
to be present? C. PTH
a. Anti-GBM antibody D. aldosterone
b. Low serum complement levels E. ADH
c. Antineutrophil cytoplasmic antibody
3. 56 y.o. woman is diagnosed w/ small cell lung
d. Elevated IgA levels
ca. she has paraneoplastic effect from the ca,
51. 440. A 28-year-old previously healthy female,
which release of an atidiruetic hormone-like
with no medical history is now 28 weeks
agent. which of the following is the most likely
pregnant. She complains of trouble seeing,
to be seen?
polyuria, polyphagia, and polydipsia. What is
her diagnosis? A. elevated serum sodium
a. Gestational diabetes mellitus B. elevated serum osmolarity
b. Deep venous thrombosis C. elevated urine sodium
c. Urinary tract infection D. elevated urine cathecolamines
d. Preeclampsia
52. 4-38. A 48-year-old man presents with
peripheral edema. He has been healthy and 4. Histological sections from an abdominal mass
physically active all of his life. His family that was removed from a 13 month old female
history is unremarkable. His blood pressure is reveal undifferentiated mesenchymal cell,
normal. On physical examination, the patient is immature tubules, and abortive glomerular
noted to have anasarca. Kidneys are not formation. What is the best diagnosis for this
palpable. Urinalysis reveals a moderate tumor?
22
A. Dupuytren meals, and located in the upper midabdomen
B. Ewing superior to umbilicus. He also report some
C. Ollier “heartburn” that has been occurring during the
D. Warthin previous year. He as been under a lot of job-
E. Wilms related stress. His stool have changed in color
over the previous 2 months and now are
intermittently dark and tarry in consistency.
5. A 63 y.o. woman has type II DM. PE positive The physician test the patient stool and find
for peripheral neuropathy in the feet and occult fecal blood. What is the most likely
nonproliferative retinopathy. Urinalysis positive diagnosis?
for proteinuria. which of the following treatment A. peptic ulcer
is most likely to attenuate the course of renal B. gastritis
disease? C. Chron’s disease
A. calcium channel blocker D. gastric polyp
B. ACE inhibitor
C. HMG-CoA inhibitor
D. dietary carbohydrate restriction 10. A 38 y.o banker with a history of heartburn
E. weight reduction suddenly experieces excruciating pain in the
epigastric region of the abdomen. Surgery is
performed immediately on admission to ER.
6. Ingesting antacids with and after a meal so There is evidence of an ulcer, which has
that gastric pH does not decrease below pH 6 ruptured through the posterior wall of
will cause a greater than normal secretion of? duodenum. What blood vessel might be
A. gastrin subject to erosion?
B. secretin A. common hepatic
C. pancreatic bicarbonate B. left gastric
D. cholecystokinin C. splenic
E. somatostatin D. superior mesenteric
E. gastroduodenal

7. A 16 y.o. girl is reffered to the office because of


chronic diarrhea and weight loss. She is KIDNEY
experiencing large volume watery diarrhea
that is painless. The symptoms persist even 54. 42 year old man, muscular weakness, mild
when she’s fasting, and there is no relationship headache after eating salted fish. Small
to foods or liquids. She’s not on any amount water drink. Usual BP 100/70, Now
medication, and there is no travel history or become 140/90, Na : 150 mEq/ml, K : 3,5,
other constitutional symptoms. Her PE is glucose 150. efek?
normal. Which of the following is the most a. Increase GFR
likely diagnosis? b. Increase obligatory water reabsorbtion
A. partial small bowel obstruction c. Increase facultative water
B. partial large bowel obstruction reabsorbtion
C. osmotic diarrhea d. Increase secretion
D. secretory diarrhea e. Increase aldotesteron secretion
E. IBS 55. To cover above function (no 1), what
mechanism
a. Increase cardiac output
8. A 42 y.o. overweight but otherwise healthy b. Increase frekuensi heart
women present with sudden onset of right c. Increase secretion rennin
upper abdominal colicky pain in 45 minutes d. Increase secretion ADH
after meal of ayam goreng. Pain is associated e. Increase secretion aldosteron
with nausea and vomiting, and any attempt to 56. Factor yang mensupport prosess diatas ialah
eat since has caused increased pain. What is a. Filtration late of glomeruli
the most likely cause? b. Active transport of tubular cell
A. gastric ulcer c. High osmolalitas of interstitium of
B. cholelithiasis medulla
C. duodenal ulcer d. Obligatory water reabsorbtion
D. acute hepatitis e. Osmalalitas of filtrate in distal
E. esophageal spasm contortus tubulus
57. Pasien dengan heart failure = low CO, perfusi
hanya organ-organ penting. Obat yang tidak
9. A 42 y.o. male, executive, complains of bole diberikan? thiazide
abdominal pain that began about 6 months
ago, is constant in nature especially after
23
58. Pasien dengan heart failure, diberi obat c. Bladder
Cloro..tiazid/mid dengan maximum dose tidak d. Uretra
merespon. Diberi apa?furosemid e. Adrenal gland
59. Pasien diberikan riazid selama… tahun. Efek 70. 70 laki-laki – came to emergency room.
yang terjadi  decrease kalium Incomplete urination since 6 bulan lalu.
60. Mountain sickness merupakan side effect? Structure yang change?
dari? a. Uretral pars membranosa
a. czetazolamide->bukan side effect, tp b. Uretral pars spongiosa
profilaksis c. Uretral pars prostatica
b. Budenamide d. Uretral orificium uretra
c. Amyloride
d. Furosemide 71. During 4 weeks treatment, the urine protein +1
e. Spironolactone , so the diagnosis :
61. Seorang pasien sedang menjalani a. Initial responder
pengobatan, pasien complaint Bright orange- b. Lat responder
red urine merupakan side effect dari obat? c. Drug resistant
a. Ethambutol d. Drug dependent
b. Isoniazid e. Remission
c. Piridizone 72. 13 years old boy comes to ER with seizure,
d. Rifampisin fever 2 weeks but has recovered, swelling in
e. Streptomisin the eyes and legs, dark urine. Unconscious ,
62. Pasien mengeluh dengan complaint(symtomp bp 180/120. Urine erythrocyte 20/hpf, protein
gonorrhea) treatmentnya? +4, albumin normal. Lab finding
a. Apnofloxacin a. ASO decrease, dnase-b increase, c3
b. Nalydixic acid decrease
c. Norifloxacin b. ASO increase, dnase-b decrease, c3
d. LEvofloxacin-- decrease
e. Ofloxacin c. ASO decrease, dnase-b increase, c3
63. Jaundice 1 yo, increase blirubinn. Ibunya increase
diberi antibiotic for UTI, baby mengalami d. ASO increase, dnase-b decrease, c3
kernicterus ,disebabkan karena obat  decrease
cotrimoxazole e. ASO increase, anti dnase-b increase,
64. 30 yo waman recurrent UTI, culture E coli di c3 decrease
kase aprofloxacin. Penghambat apa?Dna Anti-DNase B is a blood test to look for a substance
gyrase produced by Group A Streptococcus, the bacteria that
65. 30 yo man motor accident most anterior within cause strep throat.
renal sinus.  Renal vein
66. You wish examine the hilum of the left kidney 73. Underlying mechanism dari pertanyaan 19 ?
during surgery. Which pf following structure a. Streptococcal infection
must be elevated. pancreas b. Inflammation of kidney tubule
a. Stomach c. Increase permeability of glomerulus
b. Suprarenal gland membrane
c. Ascending colon d. Small pores in podocyte permit rbc
d. Duodenum pass through
e. Liver e. Heparansulfat polysaccharide is low,
67. To elevate kidney within the renal fascia dan so rbc could cross the BBB
perirenal fat which of following muscle 74. Anak di rawat karena indirect bilirubinemia. 3
a. Diafragma hari kemudian demam, dan diberi gentamicin
b. Psoas dan amoxicilin. Kenaikan BUN dan creatinine.
c. Quadratus lumborum Apa mechanisme nya?
d. Transverse abdominis a. ATN
e. Illeus b. AIN
68. Laki-laki probable testicular cancer, which c. AKI
layer is the most near to testicular parenchyma d. CKI
a. FS xterna 75. Kasus no 21 bisa terjadi karena Disebabkan
b. Tunica albuginea oleh?
c. FS interna a. Iskemik
d. Scarpa interna b. Toxic injury
e. Tunica dartos c. Drug hypersensitivity
69. 18 tahun laki-laki. Dysuria for several day – d. Immune Complex
UTI. Structure yang vulnerable get infection? 76. 10 tahun, dysuria 5 hari yang lalu. No
a. Kidney significcant finding, urine culture E coli >
b. Ureter 100.000
24
a. Acute pyelonefritis 92. Which urine specimen most appropriate?
b. Reflux nephropathy Untuk culture
c. Uncomplicated UTI a. Suprapubic
d. Complicated UTI b. Clean catch
e. Chronic UTI c. Strecth catheter
77. Which of the following factor unfavorable to d. Indualing catheter
bacterial growth in patients urine e. 24 hour urine
a. A low pH (5,5 or <) 93. UTI, beta hemolytic virulence factor
b. Low concentration urea a. Capsule
c. Abscense of organic … a diet b. P fimbrae
d. Low osmolality c. Endotoxin
e. Presence of normal microorganism d. Fosfolipase c
78. Antibiotik untuk UTI first line e. Lipopolisaccaride
a. Ceftriaxone 94. UTI, - gram, coccobacillus, high pH.virulence
b. Cefotaxime vactornya?
c. Ceftrazidime a. Hemolisin
d. Cotrimoxazole b. Fibronecrine
e. Metronidazole c. Catalase
79. Gfr <15% d. Cordially
Rifle criteria e. Opaprotein
a. Risk 95. Pada orang yang pernah mengalami atau
b. Injury sedang mengalami kidney stone,tindakan
c. Fail preventive apa yang diperlukan agar tidak
d. Loss terjadi penambahan atau recurrenci dari batu
e. ESKD ginjal tersebut?
80. Pada orang yang mengalami Hematuria in a. Restriksi air 100 ml
bladder cancer,bagaimana bentuk rasa b. Minum jus yang banyak
sakitnya  Painless and intermitten c. Minum sebanyak-banyaknya
81. PH urine = 5. Uric acid = 8,9 d. Minum pas haus
a. Lot of fluid + allopurinol e. Makan buah
b. Lots of fluid + aciding agent
82. 40 tahun old male. Thoracic gunwound, 96. 60 tahun, headache, nausea, vomit 1 bulan.
hematuria. Diagnose?renal? PE : HT, subfebrile. Pyelogram showed
83. Bladder injury. Mostly caused by?trauma affected kidney asimetri contracted with
84. Polyuria, kasih obat apa?vasopressin deform of calyceal system. Gross of the
85. Pneumocyctis carinii.. WBC increase etc.,  affected kidney showed contacted dan has
post infection glomerulonefritis irregular granular surface. Parenkim atropi and
86. Pengaruh parasimpatis terhadap bladder  replace fibrosis. Microscopi : glomerular
contraction of detrusor muscle sclerosis, hyalinisasi, atropi incortical. Another
87. Parasimpatic  S2,3,4 dan cranial 3,7,9,10 area : fibrosis, chronic inflammation cell with
88. Dementia ,cant holding a pee. frontal lobe limfosit aggregation
89. Diarrhea 8 hari what the best treatment RL a. RCC
infusion b. Pyelonefrits
90. 44 tahun laki-laki. Systemic fungi infection on c. Nefritis
amphotericin B treatment for 7 days. d. NS
Conscious, BP 120/80, pulse 80x/min, RR 16x. e. Diabetic nefropathy
36,7 c, HB 13, HT 37, WBC 8600. Platelet 97. 5 years old, pale, abdominal mass. Distention
340.000, Urinalisis RBC (0-1), WBC (2-5), to bowel e=obstraction. USG : 6 cm left renal
protein -, Granular cast +, Ureum 91, creatinin mass with necrosis and hemorrhage. Micros :
2,0, urine 800/12 jam embrional tissue with abbortive tubular and
a. ATN glomerular structure  Wilms tumor
b. Acute glomerulonefritis 98. 62 tahun, hematuri, left flank pain. PE :
c. Acute interstitial nefritis costovetebral angle tender. USG : left
d. ARF hydronefrosis. CN and CBC ….
e. CRF a. Renal biopsy
91. Woman, pain during urination and suprapubic b. R. ateriogram
pain, no fever, cloudy urine. Microscopic c. Palpasi lymph node
hematuria, pyuria, granular +, no past history d. ANA
a. Klebsiella pneumonia e. IVP
b. P. aeruginosa 99. Renal trauma, terlindas truk, ginjal kena
c. E coli kerusakan. Parenkimnya rusak. Maka yang
d. Proteus mirabilis terjadi pada ginjal tersebut ialah?
e. S. saphroliticus a. Ginjal tidak berfungsi
25
b. Ekstravasasi 3. To elevate the kidney within the renal fascia
c. Hidronefrosis and the perirenal fat, the renal fascia must be
d. Normal urogram reflected or incised from the fascia of the:
100. Injury kena renal artery a. Diaphragm
101. Pada pasien dengan pemeriksaan b. Psoas muscle
Radiology menunjukkan ada pelebaran ureter, c. Quadratus lumborum muscle
maka diperirakan batu berada di? d. Transverse abdominis muscle
a. Vesikoureter e. Iliacus muscle
b. Pelvicc ureter
102. 65, laki-laki, Ada bladder dengan 4. Which of the following is the male homologue
perbesaran prostat, kama radiografi. Gimana of the female clitoris?
 a. Epididimis
a. penonjolan dari inferior bladder b. Vas deferens
b. Penonjolan dari superior bladder c. Penis
c. Penonjolan dari superior bladder d. Scrotum
d. Penonjolan dari posterior bladder
103. 50 tahun colic. IVP = uteric dilatation. 5. The scrotum appears to have a slightly
Kenapa bisa dilatasi? pigmented and wrinkled appearance. What is
a. Stone in pelvicureter junction the explanation for this appearance?
b. Kedney cysts a. Hyperkeratinized squamous epithelium
c. Stone in the vesicoureter junction b. The tunica albugenia
d. Renal cell CA stage 1 c. The dartos fascia
104. 45 wanita, bladder a stone. d. The pampiniform plexus
Cystography
a. Indertaion of inferior aspect of bladder 6. An 18-years-old man is noted to have
b. Indertation of superior aspect of probable testicular cancer. He undergoes
bladder surgery. After incising the scrotum, the
c. Filling defect in the bladder surgeon contemplates the approach to the
parenchyma of the testes. Through which layer
must the surgeon incise to reach the testicular
1. 4yrs boy. Puffy eye in the morning,swelling leg, parenchyma?
foamy urine, protein +4, serum albumin 1,5. a. Buck’s fascia
Principle managementprednison 60mg. b. Tunica albuginea
2. Urine sample dari +4 ke +1 selama 1 bulan. c. Dartos fascia
Patient responsive classification: initial d. Scarpa’s fascia
responder
3. Boy, severe diarrhea. Not full alert. Bp 80/60, 7. A 66-year-old man complains of difficulty
pulse not palpable, RR 30, temperature voiding and is noted to have probable BPH.
38,4°C, sunken fontanele etc (moderate- Which of the following prostatic lobes is likely
severe dehydration). Distal tubular susceptible to be responsible for these symptoms?
to ischemic injury : loss of polarity re a. Anterior lobe
distribution of membrane protein b. Posterior lobe
c. Lateral lobe
1. During the removal of a patient’s kidney, you d. Middle lobe
would observe which of the following as being
most anterior within the renal sinus? 8. A 48-year-old man is undergoing cystoscopic
a. Renal arteries examination. As the cystoscope is placed into
b. Renal vein the urethra through the penile portion, which of
c. Major calyx the following tissue surrounds the urethra?
d. Minor calyx a. Prostate
e. Renal pelvis b. Corpus spongiosum
c. Seminal colliculus
2. You wish to examine the hilum of the right d. Spinchter urethrae mucles
kidney during surgery. Which of the following
structures must be elevated and reflected to 9. A police detective takes a scraping of some
do so? stains to be examined for alkaline
a. Stomach phosphatase to assess whether these might
b. Suprarenal gland be ejaculate. What is the source of alkaline
c. Ascending colon phosphatase in the semen?
d. Duodenum a. Prostatic gland
e. Liver b. Bulbourethral glands
c. Seminal vesicles
d. Seminal colliculus apparatus
26
b. Saturation of the Na-glucose cotransporter
10. A 39 yo woman complains of hematuria and c. Saturation of the Na-H exchanger
significant flank tenderness. She has a history d. Stimulation of glucose secretion
of kidney stones. A CT scan depicts the e. Stimulation of glycogen breakdown
abdominal portion of the ureter lying anterior to
a muscle. Which of the following is most likely 15. A student under stress has been feeling light-
to be the name of this muscle? headed, especially after standing, and has
a. Psoas developed a brisk diuresis. He has the smell of
b. Serratus anterior muscle acetone on his breath. Upon admission to the
c. Obturator muscle emergency room he is diagnosed with diabetic
d. Rectus muscle ketoacidosis, which is accompanied by
e. External oblique muscle extreme hypovolemia, supposedly because of
the brisk diuresis. The brisk diuresis is a
11. An individual is known to be suffering from consequence of which of the following?
DM. Recently, he has developed hypertension. a. High levels of glucose in the tubular
His doctor suspects that the patient may be fluid/urine
developing renal insufficiency that is leading to b. Increased GFR
a reduced glomerular filtration and, as a result, c. Suppression of ADH secretion
hypervolemia and hypertension. The doctor d. Suppression of aldosterone secretion
wishes to evaluate kidney function by e. Decreased angiotensin II plasma levels.
measuring the GFR. Which is the best
substances for estimating the GFR from the 16. A hypertensive patient was placed on the
urine? diuretic Lasix (furosemid) to increase urinary
a. Creatinine output. Furosemid, a “high-ceiling” diuretic, is
b. Para-aminohippuric acid (PAH) a potent diuretic because it binds to and
c. Urea inhibits which of the following transport
d. Glucose processes?
e. Sodium a. The Na-glucose co transporter in the
proximal tubule
12. A 21 yo man has been vomiting to the point b. The Na-K exchange pump in all nephron
where he has become hypovolemic, as segments
evidence by an accompanying decrease in c. The Na-K-Cl cotransporter in the thick
blood pressure and a feeling of light- ascending limb
headedness. The kidneys respond by reducing d. The Na-Cl cotransporter in the distal
urinary volume flow, thus limiting the potential convoluted tubule
extent of hypovolemia. Increase in the plasma e. The Na channel in the cortical collecting
levels of which of the following hormones will duct.
bring about the most dramatic decrease in
urinary volume flow rate? 17. A hypertensive patient is prescribed a loop
a. Angiotensin II diuretic such as Lasix without any
b. Atrial natriuretic peptide supplements. One week later the patient
c. PTH returns to the clinic complaining of dizziness,
d. Aldosterone weakness, and nausea. The most likely cause
e. ADH of the patients worsening condition is the
development of :
13. A 56 yo woman is diagnosed with small cell a. Metabolic acidosis
carcinoma of the lung. She has a b. Hyponatremia
paraneoplastic effect from the cancer, with c. Hypocalcaemia
release of an antidiuretic hormone-like agent. d. Hypokalemia
Which of the following is the most likely to be e. Hypovolemia
seen?
a. Elevated serum sodium 18. A 35 yo female is noted to have new onset
b. Elevated serum osmolarity hypertension that is thought to be due to an
c. Elevated urine sodium aldosterone-secreting adrenal tumor. Which of
d. Elevated urine catecholamines the following is likely to be seen in this patient?
a. Hypertension markedly improved with
14. An individual has adult-onset diabetes. She furosemid
has high levels of glucose in the urine and is b. Elevated serum sodium level
experiencing a brisk diuresis. The appearance c. Elevated serum potassium level
of glucose in the urine is a consequence of d. Elevated urinary cortisol level
which of the following processes in the
proximal tubules? 19. A renal biopsy from an adult who presented
a. Inhibition of Na-K ATPase (Na+ pump) with progressive renal failure and hematuria
27
reveals linear deposits of IgG within the a. Dupuytren tumor
glomeruli. What type of autoantibody is most b. Ewing tumor
likely to be present in this individual? c. Ollier tumor
a. Anti-basement membrane antibodies d. Warthin tumor
b. Anti-centromere antibodies e. Wilms tumor
c. Anti-double-stranded DNA antibodies
d. Antimitochondria antibodies 24. A 69 yo man has lost a friend to prostate
e. Anti-smooth muscle antibodies cancer, and would like to be evaluated for the
disease. He has no urinary symptoms. Which
20. Which one of the following combinations of of the following test is most likely indicated to
signs and symptoms is most consistent with a screen him for prostate cancer?
diagnosis of nephritic syndrome? a. Prostate ultrasound
a. Hematuria, hypertension, and proteinuria b. Digital Rectal Examination (DRE)
b. Massive proteinuria, edema, and c. DRE and prostate specific antigen (PSA)
hyperlipidemia d. PSA
c. Oliguria, hydronephrosis, and abdominal e. None of above
rebound tenderness
d. Painful hematuria, flank pain, and palpable 25. A 63 yo woman has Type II DM, which is well-
abdominal mass controlled. Her physical examination is positive
e. Painless hematuria, polycythemia, and for peripheral neuropathy in the feet and
increased skin pigmentation nonproliferative retinopathy. A urinalysis is
positive for proteinuria. Which of the following
21. A 30 yo female patient presents with a new treatment is most likely to attenuate the course
onset of peripheral edema. PE finds of renal disease?
hypertension & bilateral pedal edema. a. Calcium channel blockers
Urinalysis finds massive proteinuria, and b. ACE inhibitors
evaluation of her serum finds elevated levels c. Hepatic hydroxymethylglutaryl coenzyme
of cholesterol. A silver stain of a renal biopsy A (HMG-CoA) inhibitors
specimen reveals a characteristic “spike and d. Dietary carbohydrate restriction
dome pattern, and electron microscopy finds a e. Weight reduction
uniform deposition of small electron dense 26. A 22 yo woman come to the clinic with sign of
deposits in a subepithelial location. Which one septic shock. The doctor prescribe her to
of the following immunofluorescence patterns consume antibiotic for 2 weeks. After one
is most characteristic of this patient’s renal week taking the medications she come to the
disease? doctor with complain of decrease in urine
a. Granular pattern of IgA and C3 volume, and edema. On physical examination
b. Granular pattern of IgG and C3-acute found that the blood pressure is 140/90, heart
glomerulonephritis rate 80, and RR=20. The urinalysis show
c. Linear pattern of IgD and C4 proteinuria and muddy brown cast were found.
d. Linear pattern of IgE and C4 What is the most possible antibiotic that can
e. Linear pattern of IgM and C3 cause this symptoms?
a. Amoxycillin
22. A 54 yo man presents with worsening pain on b. ceftriaxone
the left side accompanied by gross hematuria. c. cefixim
Workup finds a 4.5 cm pas in the upper pole of d. Levofloxacin
the left kidney. What is the characteristic e. Gentamycin
histological appearance of the conventional
type of malignancy in this location? 27. A 69 yo woman present with left flank pain and
a. Disorganized groups of immature tubules hematuria. Physical examination suggest a left
b. Sheets of transitional epithelial cells side abdominal mass. CT scan of abdomen
c. Small cells forming numerous papillary reveal 5cm mass in the left kidney. Which of
structures the following laboratory abnormalities might
d. Undifferentiated cells demonstrating also be present?
abortive glomerular formation a. Polycythemia
e. Uniform cells with clear cytoplasm b. Thrombocytopenia
resulting from glycogen c. Hypocalcemia
d. Leukocytosis
23. Histological sections from an abdominal mass e. High rennin hypertension
that was removed from a 13-month-old female
reveal undifferentiated mesenchymal cells, 28. A 60 yo man with heart failure and normal
immature tubules, and abortive glomerular renal function is start of furosemid (Lasix)
formation. What is the best diagnosis for this 80mg/day. She notice a good diuretic
tumor? response every time she take the medication.
28
A few week later, she is feeling unwell
because of muscle weakness and fatigue, but
her heart failure are better. Which of the
following is the most likely explanation for her
muscle weakness?
a. Hyponatremia
b. Hypernatremia
c. Hypokalemia
d. Hyperkalemia
e. Anemia

29. A 78 yo man is brought to the hospital


because of nausea and vomiting. On the
physical examination he appear dry, his
abdominal is soft, the JVP is not visible. His
laboratory test reveal hypernatremia and free
water deficit is approximately 3L. In what part
of the normal kidney is most of the water
reabsorb from?
a. Colleting duct
b. Proximal tubulus
c. Distal tubulus
d. Ascending loop of Henle
e. Descending loop of Henle

30. A 25 yo woman present with pain on his back


and abdomen. She also complain of having
fever, nausea and vomiting. About 1 year ago
she has history of renal stone and sometimes
she found some solid material in her urine. On
histopathology examination found
glomerulosclerosis, thickening of the artery,
granulation tissue, and limfosit. What is your
working diagnosis according to the
histopathology?
a. Nephroblastoma
b. Chronic pyelonephritis
c. Acute tubular necrosis
d. Cystitis
e. Acute pyelonephritis

31. A 63 yo man with an 8 year history of recurrent


severe arthritis in his large toes has dysuria.
He also had pain that comes from his back
that is radiating to the scrotum. Which of the
following mechanism is the most likely
explanation for his symptoms?
a. Uric acid kidney stones
b. ARF
c. Renal parenchymal uric acid crystals
d. Chronic pyelonephritis
e. Acute pyelonephritis

29

Вам также может понравиться